0% found this document useful (0 votes)
437 views65 pages

Merchandising - Review Materials (Problems)

The document provides examples and questions related to calculating costs of merchandise under different credit terms and purchase scenarios using both the gross and net methods. Specifically: - It gives examples of different credit terms (e.g. 2/10, n/30) and calculates implied annual interest rates and discounts. - Questions calculate inventory costs under the gross method accounting for list prices, trade discounts, and credit terms. - Other questions calculate inventory costs under the net method by taking the invoice price and adding freight-in, insurance, tariffs and other costs while subtracting purchase returns. - Cost of merchandise is the key figure to record for inventory valuation and financial reporting.

Uploaded by

juls
Copyright
© © All Rights Reserved
We take content rights seriously. If you suspect this is your content, claim it here.
Available Formats
Download as DOCX, PDF, TXT or read online on Scribd
0% found this document useful (0 votes)
437 views65 pages

Merchandising - Review Materials (Problems)

The document provides examples and questions related to calculating costs of merchandise under different credit terms and purchase scenarios using both the gross and net methods. Specifically: - It gives examples of different credit terms (e.g. 2/10, n/30) and calculates implied annual interest rates and discounts. - Questions calculate inventory costs under the gross method accounting for list prices, trade discounts, and credit terms. - Other questions calculate inventory costs under the net method by taking the invoice price and adding freight-in, insurance, tariffs and other costs while subtracting purchase returns. - Cost of merchandise is the key figure to record for inventory valuation and financial reporting.

Uploaded by

juls
Copyright
© © All Rights Reserved
We take content rights seriously. If you suspect this is your content, claim it here.
Available Formats
Download as DOCX, PDF, TXT or read online on Scribd
You are on page 1/ 65

BASIC CONCEPTS

Credit Terms
Annualized percentage rate
50. Which of the following credit terms is the most advantageous to the purchaser of merchandise?
A. 1/10, n/30. C. 2/10, n/30.
B. 5/10, n/60. D. 5/10, n/20.

64. Taking advantage of a 2/10, n/30 purchases discount is equal to a savings yearly rate of approximately (E)
a. 2% c. 20%
b. 24% d. 36%
i
. A buyer failed to take advantage of the vendor’s credit terms of 2/15, n/45, but instead paid the invoice in full at
the end of 60 days. By not taking advantage of the cash discount, the buyer lost the equivalent of
____________ annual interest on the amount of the purchase. (D)
A. 12.2% D. 24.3%
B. 16.2% E. 24.5%
C. 18.9%

66. Stine Company purchased merchandise with an invoice price of $2,000 and credit terms of 2/10, n/30. Assuming
a 360 day year, what is the implied annual interest rate inherent in the credit terms?
a. 20% c. 36%
b. 24% d. 72%

Purchases – Gross Method


Invoice price
Net of trade discount
ii
. Ami Retailers purchased merchandise with a list price of $100,000, subject to a trade discount of 20 percent and
credit terms of 2/10, n/30. At what amount should Ami record the cost of this merchandise if the gross method is
used? (E)
a. $100,000 c. $98,000
b. $80,000 d. $78,400 S, S & S

48. A retailer purchases merchandise with a catalog list price of $10,000. The retailer receives a 25% trade discount
and credit terms of 2/10, n/30. What amount should the retailer debit to the Merchandise Inventory account? (E)
a. $7,500 c. $9,800
b. $10,000 d. $7,350
iii
. West Retailers purchased merchandise with a list price of $20,000, subject to trade discounts of 20% and 10%,
with no cash discounts allowable. West should record the cost of this merchandise as (E)
a. $14,000 c. $15,600
b. $14,400 d. $20,000 AICPA 0590
iv
. Utley Retailers purchased merchandise with a list price of $30,000, subject to trade discounts of 20% and 10%,
with no cash discounts allowable. Utley should record the cost of this merchandise as (E)
a. $21,000. c. $23,400.
b. $21,600. d. $30,000. K, W & W
v
. MORO Co. purchased an item of merchandise quoted and listed at P150,000 under the following terms: Trade
discounts of 15%, 10% and 5%; 2/10, n/30. What was the invoice price of the merchandise? (E)
a. P105,000.00 c. P100,842.00
b. P109,012.50 d. P104,695.60 RPCPA 1087
Discount taken
Paid within discount period
18. Jill buys $775 of merchandise on account from Toys Are Fun. Her customer terms are 3/10, n/45. The amount
of her discount if she pays within the discount period is:
A. $ 77.50. C. $697.50.
B. $ 23.25. D. $ 0. 00

Paid within discount period, with purchase return


24. ABC Company returned $750 of goods it had purchased from another company. The original invoice was for
$4,200, 3/10, n/30. What is the discount if ABC pays the balance within the discount period?
A. $ 126.00 C. $ 22.50
B. $ 103.50 D. $ 0.00

Purchases – Net Method


Inventoriable costs
Invoice price
vi
. The following costs were among those incurred by Woodcroft Corporation during 2011:
Merchandise purchased for resale $500,000
Salesmen's commissions 40,000
Interest on notes payable to vendors 5,000
How much should be charged to the cost of the merchandise purchases? (E)
A: $500,000 C: $540,000
B: $505,000 D: $545,000 Wiley 2011
vii
. On December 28, 2001, Nord Manufacturing Co. purchased goods costing $50,000. The terms were FOB
destination. Some costs incurred in connection with the sale and delivery of the goods were
Packaging for shipping $1,000
Shipping 1,500
Special handling charges 2,000
These goods were received on December 31, 2001. In Nord’s December 31, 2001 balance sheet, what amount
of cost should be included in inventory? (E)
A. $54,500 C. $52,000
B. $53,500 D. $50,000 AICPA 1191
viii
. On December 26, 2004, Karen Company purchased goods costing P5,000,000. The freight term is FOB
destination. Some of the costs incurred in connection with the sale and delivery if the goods were:
Packaging for shipment 100,000
Shipping 200,000
Special handling charges 300,000
These goods were received on December 31, 2004. In the December 31, 2004 balance sheet, what amount of
cost for these goods should be included in inventory? (E)
A. 5,000,000 C. 5,300,000
B. 5,600,000 D. 5,500,000 CPAR

7. On December 28, 1996, Arlington Co. purchased goods costing P1,000,000. The terms were FOB destination.
Some of the cost incurred in connection with the sale and delivery of the foods were as follows:
Packaging for shipment P20,000
Shipping 30,000
Special handling charges 40,000
These goods were received on December 31, 1996.
In Arlington’s December 31, 1996 balance sheet, what amount of costs for these goods should be included in
inventory? (E))
A. P1,040,000 C. P1,090,000
B. P1,070,000 D. P1,000,000 RPCPA 1096

List price – trade discount + freight-in


ix
. Dean Sportswear regularly buys sweaters from Mill Company and is allowed trade discounts of 20% and 10%
from the list price. Dean made a purchase on March 20, 2004, and receive an invoice with a list price of
P600,000, a freight charge of P15,000 and payment terms of 2/10, n/30. Dean should record the purchase at
(E)
A. 432,000 C. 438,360
B. 447,000 D. 435,000 AICPA

Invoice price + insurance + tariff costs


78. If the merchandise costs $4,000, insurance in transit costs $200, tariff costs $50, processing the purchase order
by the purchasing department costs $35, and the company receiving dock personnel cost $15, what is the total
cost charged to the merchandise? (D)
a. $4,250 c. $4,300
b. $4,285 d. $4,000

Invoice price + freight-in – purchase return,


57. The following information applied to Greer, Inc. for 2001:
Merchandise purchased for resale $200,000
Freight-in 8,000
Freight-out 5,000
Purchase returns 2,000
Greer's 2001 inventoriable cost was (E)
A. $200,000. C. $206,000.
B. $203,000. D. $211,000. K, W & W
x
. The following information applied to Fenn, Inc. for 1989:
Merchandise purchased for resale $400,000
Freight-in 10,000
Freight-out 5,000
Purchase return 2,000
Fenn’s 1989 inventoriable cost was (E)
A. $400,000 C. $408,000
B. $403,000 D. $413,000 AICPA 1190
xi
. The following information applied to Atlas Co. for 2001:
Merchandise purchased for resale $800,000
Freight-in 20,000
Freight-out 10,000
Purchase returns 4,000
The company’s 2001 inventoriable cost was (E)
A. $800,000 C. $816,000
B. $806,000 D. $826,000 AICPA 1190 I-7

95. The following information applied to Landon Company for 2005:


Merchandise purchased for resale ..................... $300,000
Freight-in ........................................... 7,500
Interest on notes payable to vendors ................. 3,000
Purchase returns ..................................... 1,500
Landon's inventoriable cost for 2005 was (E)
A. $309,000. C. $306,000.
B. $307,500. D. $301,500. S, S & S

Purchases + freight-in -- purchase discount


145.West Company has the following account balances:
Purchases $48,000
Sales Returns and Allowances 6,400
Purchase Discounts 4,000
Freight-in 3,000
Delivery Expense 4,000
The cost of goods purchased for the period is
a. $52,000. c. $51,000.
b. $47,000. d. $44,600.

Purchases + freight-in – purchase returns & allowances -- purchase discounts


6. A company using the periodic inventory system has the following account balances: Merchandise Inventory at
the beginning of the year, $4,000; Transportation-In, $450; Purchases, $12,000; Purchases Returns and
Allowances, $2,300; Purchases Discounts, $220. The cost of merchandise purchased is equal to (M)
a. $13,930 c. $9,489
b. $9,930 d. $14,520

69. Zach’s Market recorded the following events involving a recent purchase of merchandise:
 Received goods for $50,000, terms 2/10, n/30.
 Returned $1,000 of the shipment for credit.
 Paid $250 freight on the shipment.
 Paid the invoice within the discount period.
As a result of these events, the company’s merchandise inventory
a. increased by $48,020. c. increased by $48,265.
b. increased by $49,250. d. increased by $48,270.

70. Jake’s Market recorded the following events involving a recent purchase of merchandise:
 Received goods for $20,000, terms 2/10, n/30.
 Returned $400 of the shipment for credit.
 Paid $100 freight on the shipment.
 Paid the invoice within the discount period.
As a result of these events, the company’s merchandise inventory
a. increased by $19,208. c. increased by $19,306.
b. increased by $19,700. d. increased by $19,308.

FOB Shipping point


xii
. An enterprise with a December 31 year-end purchased 2,000 of inventory on account. The seller was
responsible for delivery to the shipping point, with freight of 50 paid at destination by the buyer. The invoice date
was December 27, 2001, and the goods arrived on January 3, 2002. What is the correct amount of inventory
and freight-in, respectively, relating to this purchase on the 2001 financial statements?
CIA 0592 IV-32 A. B. C. D.
Inventory 0 2,050 0 2,000
Freight-In 0 0 50 50
FOB destination
xiii
. An enterprise with a December 31 year-end purchased 2,000 of inventory on account. The terms required the
seller to deliver to the destination, with freight of 50 paid at destination by the buyer. The invoice date was
December 27, 2001, and the goods arrived on January 3, 2002. What is the correct amount of inventory and
freight-in relating to this purchase on the 2001 financial statements?
CIA 0592 IV-33 A. B. C. D.
Inventory 0 2,050 0 2,000
Freight-In 0 0 50 50

Freight-in +handling costs + insurance on shipment


xiv
. On December 15, 2011, Flanagan purchased goods costing $100,000. The terms were FOB shipping point.
Costs incurred by Flanagan in connection with the purchase and delivery of the goods were as follows:
Normal freight charges $3,000
Handling costs 2,000
Insurance on shipment 500
Abnormal freight charges for express shipping 1,200
The goods were received on December 17, 2011. What is the amount that Flanagan should charge to inventory
and to current period expense? (E)
A B C D
Inventory $3,000 $5,000 $5,500 $6,700
Current period $3,700 $1,700 $1,200 $0
expense

Payment to Suppliers
Within discount period
Net of purchase discount
xv
. A firm purchased goods with a purchase price of $1,000 and credit terms of 1/10 net 30. The firm paid for these
goods on the 5th day after the date of sale. The firm must pay _____ for the goods. (E)
A. $990 C. $1,000
B. $900 D. $1,100 Gitman
xvi
. A firm purchased goods on January 27 with a purchase price of $1,000 and credit terms of 2/10 net 30 EOM.
The firm paid for these goods on February 9. The firm must pay _____ for the goods. (E)
A. $1,000 C. $800
B. $980 D. $900 Gitman

15. The amount of an invoice is $1000, with terms 2/10, n30. The amount to be paid within the discount period is (E)
A. $1,000 C. $ 900
B. $ 980 D. $ 700

17. An invoice of $237.50 is dated April 2, terms 2/10, n/30. If the invoice is paid on April 9, the amount to be paid is
(E)
A. $ 4.75. C. $232.75.
B. $ 23.75. D. $237.50.

65. Flynn Company purchased merchandise inventory with an invoice price of $5,000 and credit terms of 2/10, n/30.
What is the net cost of the goods if Flynn Company pays within the discount period? (E)
a. $5,000 c. $4,500
b. $4,900 d. $4,600
55. Merchandise with an invoice price of $4,000 is purchased on June 2 subject to terms of 2/10, n/30, FOB
destination. Transportation costs paid by the seller totaled $150. What is the cost of the merchandise if paid on
June 12, assuming the discount is taken? (M)
a. $4,150 c. $4,067
b. $4,070 d. $3,920

Net of trade discount & purchase discount


73. A retailer purchases merchandise with a catalog list price of $10,000. The retailer receives a 25% trade discount
and credit terms of 2/10, n/30. How much cash will be needed to pay this invoice within the discount period? (E)
a. $10,000 c. $9,800
b. $7,500 d. $7,350
xvii
. The list price and term of a merchandise purchased is P8,000 less 15%, 10%, 5%, 2/10, n/30. The payment
within the discount period is (E)
a. P5,697.72 c. P5,488.
b. P5,654.00 d. P7,840 RPCPA 0587
xviii
. MORO Co. purchased an item of merchandise quoted and listed at P150,000 under the following terms: Trade
discounts of 15%, 10% and 5%; 2/10, n/30. How much was the payment if paid within the discount period? (E)
a. P102,900.00 c. P100,842.00
b. P106,832.25 d. P104,695.60 RPCPA 1087

Net of trade discount & purchase discount + prepaid freight


45. On June 1, 1997, Ebecom Corp. sold merchandise with a list price of P200,000 to Greenwich, Inc. on account.
Ebecom allowed trade discounts of 20% and 10%. Credit terms were 2/30, n/60 and the sale was made FOB
shipping point. Ebecom prepaid P3,000 of insurance for Greenwich as an accommodation.
On June 20, 1997, Greenwich paid Ebecom his full account of (E)
a. P150,060 c. P147,060
b. P144,120 d. P141,120 RPCPA 0597

26. On June 1, 1995, Orange Corp. sold merchandise with list price of P5,000 to Mont on account. Orange allowed
trade discounts of 30% and 20%. Credit terms were 2/15, n/40 and the sale was made FOB shipping point.
Orange prepaid P200 of delivery costs for Mont as an accommodation. On June 12, 1995, Orange received
from Month a remittance in full payment amounting to (M)
a. P2,744 c. P3,140
b. P2,940 d. P2,944 RPCPA 0596

Net of purchase returns & purchase discount


xix
. A company purchased $1,800 of merchandise on December 5. On December 7, it returned $200 worth of
merchandise. On December 8, it paid the balance in full, taking a 2% discount. The amount of the cash paid on
December 8 equals: (E)
A. $ 200. D. $1,600.
B. $1,564. E. $1,800.
C. $1,568.
xx
. Harvest Food Stores purchased canned goods at an invoice price of $3,000 and terms of 2/10, n/30. Half of the
goods had been mislabeled and were returned immediately to the supplier. If Harvest Food pays the remaining
amount of the invoice within the discount period, the amount paid should be (E)
A. $1,440. C. $2,940.
B. $1,470. D. $3,000.
Net of purchase return, purchase discount + freight-in
xxi
. A company purchased $4,000 worth of merchandise. Transportation costs were an additional $350. The
company later returned $275 worth of merchandise and paid the invoice within the 2% cash discount period.
The total amount paid for this merchandise is: (E*)
A. $3,725.00. D. $4,000.50.
B. $3,925.00. E. $4,075.00.
C. $3,995.00.

After discount period


Invoice price
89. If Fabulous Furniture, Inc purchased inventory at $900 list price and the terms were 3/10 n/30, what would be the
value associated with the inventory if payment was made after 20 days? (E)
A. $700. C. $1000.
B. $970. D. $900.

Invoice price – prepaid freight


xxii
. Sylvia Systems Corp. received an invoice for P12,000 from Lyndon Enterprises Co. for a shipment of specialty
supplies, F.O.B. destinations, terms 2/10, n/60. In addition, the Sylvia Systems paid P800 for freight on this
shipment to a trucking firm. If the invoice from Lyndon Enterprises is paid 15 days after it is received, Sylvia
Systems must pay (E)
a. P10,960 d. P10,976
b. P11,200 e. P12,544
c. P11,184 RPCPA 1078

Sales – Gross Method


Partial delivery
xxiii
. On October 1, 2000, Acme Fuel Co. sold 100,000 gallons of heating oil to Karn Co. at $3 per gallon. Fifty
thousand gallons were delivered on December 15, 2000, and the remaining 50,000 gallons were delivered on
January 15, 2001. Payment terms were 50% due on October 1, 2000, 25% due on first delivery, and the
remaining 25% due on second delivery. What amount of revenue should Acme recognize from this sale during
2000?
A. $75,000 C. $225,000
B. $150,000 D. $300,000 AICPA 0592

Discount granted
Within discount period, prepaid freight, credit memo
36. Merchandise subject to terms 1/10, n/30, FOB shipping point, is sold on account to a customer for $15,000. The
seller paid transportation costs of $1,000 and issued a credit memorandum for $5,000 prior to payment. What is
the amount of the cash discount allowable? (D)
a. $160 c. $140
b. $150 d. $100

After discount period


27. First Company sold merchandise on credit to Second Company for $1,000 on July 1, with terms of 2/10, net /30.
On July 6, Second returned $200 worth of merchandise claiming the materials were defective. On July 8, First
received a payment from Second and credited Accounts Receivable for $450. On July 24, Second Company
paid the remaining balance on its account. How much was the total Sales Discounts given to Second during
July?
a. $0 c. $441
b. $9 d. $2,441 S, S & S
Sales – Net Method
Cash collection
Within discount period
Net of sales discount
74. Company X sells $400 of merchandise on account to Company Y with credit terms of 2/10, n/30. If Company Y
remits a check taking advantage of the discount offered, what is the amount of Company Y's check?
a. $280 c. $360
b. $392 d. $320

94. Company A sells $500 of merchandise on account to Company B with credit terms of 2/10, n/30. If Company B
remits a check taking advantage of the discount offered, what is the amount of Company B's check?
a. $350 c. $450
b. $490 d. $400

. Gibbson Company sold inventory that cost $4,000 to Garrison Company for $6,000. The inventory was sold
under the terms 1/10, n/30. The receivable was collected within the discount period. The goods were delivered
FOB destination and freight costs of $200 were paid in cash. Based on this information alone (and assuming
that the company uses a perpetual inventory system), what amount would Gibbson Company collect from
Garrison Company?
A. $2,000 C. $6,000
B. $5,940 D. $6,140

Net of sales return & discount


71. A credit sale of $800 is made on April 25, terms 2/10, n/30, on which a return of $50 is granted on April 28.
What amount is received as payment in full on May 4?
a. $735 c. $800
b. $784 d $750

75. Holt Company sells merchandise on account for $2,000 to Jones Company with credit terms of 2/10, n/30. Jones
Company returns $400 of merchandise that was damaged, along with a check to settle the account within the
discount period. What is the amount of the check?
a. $1,960 c. $1,600
b. $1,968 d. $1,568

87. A credit sale of $900 is made on July 15, terms 2/10, n/30, on which a return of $50 is granted on July 18. What
amount is received as payment in full on July 24?
a. $900 c. $850
b. $833 d $882

95. Hale Company sells merchandise on account for $1,500 to Kear Company with credit terms of 2/10, n/30. Kear
Company returns $300 of merchandise that was damaged, along with a check to settle the account within the
discount period. What is the amount of the check?
a. $1,470 c. $1,200
b. $1,476 d. $1,176

27. Silver Co. sold merchandise to Bronze Co. on account, $23,000, terms 2/15, net 45. The cost of the
merchandise sold is $18,500. Silver Co. issued a credit memorandum for $2,500 for merchandise returned that
originally cost $1,900. The Bronze Co. paid the invoice within the discount period. What is amount of net sales
from the above transactions? (D)
a. $20,090 c. $3,490
b. $20,500 d. $23,000

Net of sales discount + freight-in


. Gibbson Company sold inventory that cost $4,000 to Garrison Company for $6,000. The inventory was sold
under the terms 1/10, n/30. The receivable was collected within the discount period. The goods were delivered
FOB shipping point and freight costs of $200 were paid in cash. Based on this information alone, what would the
inventory value be on Garrison Company’s balance sheet?
A. $2,000 C. $6,000
B. $5,940 D. $6,140

Net of sales return, sales discount + prepaid freight,


49. A sales invoice included the following information: merchandise price, $4,000; transportation, $300; terms 1/10,
n/eom, FOB shipping point. Assuming that a credit for merchandise returned of $600 is granted prior to payment,
that the transportation is prepaid by the seller, and that the invoice is paid within the discount period, what is the
amount of cash received by the seller? (D)
a. $3,366 c. $3,666
b. $3,400 d. $3,950

Net of trade discount, sales discount + prepaid freight


xxiv
. On June 1, 2001, Oslo Corp. sold merchandise with a list price of $15,000 to Mead on account. Oslo allowed
trade discounts of 30% and 20%. Credit terms were 2/15, n/40 and the sale was made f.o.b. shipping point. Oslo
prepaid $300 of delivery costs for Mead as an accommodation. On June 12, 2001, Oslo received from Mead a
remittance in full payment amounting to (E)
a. $8,232. c. $8,532.
b. $8,526. d. $8,397. K, W & W
xxv
. On June 1, 2003 Taal Company sold merchandise with a list price of P5,000,000 to XYZ. Taal allowed trade
discounts of 20% and 10%. Credit terms were 5/10, n/30 and the sale was made FOB shipping point. Taal
prepaid P200,000 of delivery cost for XYZ as an accommodation. On June 11, 2003. Taal received from XYZ full
remittance of (M)
A. 3,420,000 C. 3,600,000
B. 3,620,000 D. 3,800,000 CPAR 4126
xxvi
. On June 1, 2001, Halle Corp. sold merchandise with a list price of $5,000 to Bonn on account. Halle allowed
trade discounts of 30% and 20%. Credit terms were 2/15, n/40, and the sale was made FOB shipping point.
Halle prepaid $200 of delivery costs for Bonn as an accommodation. On June 12, 2001, Halle received from
Bonn a remittance in full payment amounting to (M)
A. $2,744 C. $2,944
B. $2,940 D. $3,140 AICPA, adapted

After discount period

zzz
PERPETUAL INVENTORY SYSTEM
Comprehensive
Questions 65 thru 70 are based on the following information.
Based on the following information,
1. $5,000 of merchandise inventory was ordered on April 2, 2007
2. $2,000 of this merchandise was received on April 5, 2007
3. On April 6, 2007, an invoice dated April 4, 2007, with terms of 2/10, net 30 for $2,150 which included a $150
prepaid freight cost, was received.
4. On April 10, 2007, $500 of the merchandise was returned to the seller.

68. The entry for April 10, 2007 would include? (D)
a. Debit to Merchandise Inventory $500 c. Credit to Merchandise Inventory $500
b. Debit to Purchases Returns $500 d. Credit to Accounts Payable $500

65. What would be recorded as purchases discount if the invoice is paid within the discount period? (D)
a. $100 c. $43
b. $30 d. $33

66. What would be recorded as the cash payment if the invoice is paid within the discount period? (D)
a. $1,470 c. $2,150
b. $1,520 d. $1,620

67. What would be recorded as net purchases amount after all of the transactions have been recorded? (D)
a. $2,000 c. $1,620
b. $2,150 d. $1,470

69. By what date does the invoice need to be paid in order to take the advantage of the discount? (D)
a. April 15, 2007 c. April 10, 2007
b. April 16, 2007 d. April 14, 2007

70. What would be the cash payment if the company decides to pay the invoice on April 30, 2007? (M)
a. $1,650 c. $2,150
b. $1,620 d. $2,000
i
. (365 / [45-15]) x .02 = 24.3%
ii
.$80,000 ($100,000 x 80%)
iii
.REQUIRED: The amount to be recorded as cost of inventory subject to trade discounts.
DISCUSSION: (B) When inventory is subject to cash discounts, the purchases may be reflected either net of these
discounts or at the gross prices. However, purchases should always be recorded net of trade discounts. A chain
discount is the application of more than one trade discount to a list price. Chain discounts should be applied in steps as
indicated below.
List price $20,000
20% discount (4,000)
$16,000
10% discount (1,600)
Cost of merchandise $14,400
Answer (A) is incorrect because $14,000 applies both discounts to the retail price. Answer (C) is incorrect because $15,600
assumes the 10% discount is applied to the 20% discount. Answer (D) is incorrect because $20,000 is the list price, and it
fails to reflect the discounts.
iv
.$21,600 ($30,000 x 80% x 90%)
v
.P109,012,50 (P150,000 x 85% x 90% x 95%)
vi
.Answer A is correct. The costs to be charged to merchandise purchases should include those costs necessary to
prepare the merchandise for sale. Salesmen's commissions are a selling expense and not related to the acquisition of
the merchandise. These costs are expensed in the period incurred. The interest is a financing expense and is also
expensed in the period incurred. Thus, only the $500,000 should be included in the cost of the merchandise purchases.
vii
.REQUIRED: The amount of cost for goods included in inventory.
DISCUSSION: (D) FOB destination means that title passes upon delivery at the destination, the seller bears the risk of
loss, and the seller is responsible for the expense of delivering the goods to the designated point. Consequently, the
packaging, shipping, and handling costs are not included in the inventory. The amount that should be included is
therefore the purchase price of $50,000.
Answer (A) is incorrect because the packaging, shipping, and handling costs should not be included. Answer (B) is
incorrect because the shipping and handling costs should not be included. Answer (C) is incorrect because the handling
costs should not be included.
viii
.A Cost of inventory 5,000,000
ix
.List price 600,000
Trade discounts (20% x 600,000) (120,000)
Balance 480,000
(10% x 480,000) ( 48,000)
Invoice price 432,000
Freight charge 15,000
Total cost of purchase 447,000

Purchases are normally recorded at gross. Thus, the cash discount is ignored.
x
.REQUIRED: The amount of inventoriable cost for the year.
DISCUSSION: (C) Inventoriable cost is the sum of the applicable expenditures and charges directly or indirectly
incurred in bringing all items of inventory to their existing condition and location. Thus, inventoriable cost includes the
$400,000 cost of the merchandise purchased, plus the $10,000 of freight-in, minus the $2,000 of purchase returns.
Freight-out is not a cost incurred in bringing the inventory to a salable condition. Consequently, the inventoriable cost
for Fenn was 4408,000 ($400,000 + $10,000 – $2,000).
Answer (A) is incorrect because $400,000 excludes freight-in and purchase returns. Answer (B) is incorrect because
$403,000 excludes freight-in. Answer (D) is incorrect because $413,000 includes freight-out.
xi
.REQUIRED: The amount of inventoriable cost.
DISCUSSION: (C) Inventoriable cost is the sum of the applicable expenditures and charges directly or indirectly
incurred in bringing all items of inventory to their existing condition and location. Thus, inventoriable cost includes the
$800,000 cost o the merchandise purchased, plus the $20,000 of freight-in, minus the $4,000 of purchase returns.
Freight-out is not a cost incurred in bringing the inventory to a salable condition. The inventoriable cost for Atlas during
2001 is $816,000 ($800,000 + $20,000 - $4,000)
Answer (A) is incorrect because $800,000 is the amount of gross purchases. Answer (B) is incorrect because $806,000
incorrectly includes freight-out as a cost instead of freight-in. Answer (D) is incorrect because $826,000 incorrectly
includes freight-out.
xii
.Answer (D) is correct. The shipping term indicates that title and risk of loss passed to the buyer at
the shipping point. Hence, the 2001 ending inventory should include the 2,000 cost of this
purchase. Also, the buyer is responsible for 50 of freight regardless of which party initially paid.
The seller bears the expense of delivery to the shipping point, not the destination.
Answer (A) is incorrect because inventory and freight-in should be 2,000 and 50, respectively. Answer (B) is incorrect
because inventory and freight-in should be 2,000 and 50, respectively. Answer (C) is incorrect because inventory
should be 2,000.
xiii
.Answer (A) is correct. Title and risk of loss passed to the buyer at the destination, and the seller incurred the expense of
delivery to that point. The goods did not arrive until after year-end, so they should not be included in 2001 inventory.
Freight-in should also not be recorded until 2002.
Answer (B) is incorrect because no inventory should be included in the 2001 financial statements. Answer (C) is
incorrect because freight-in should be recorded in 2002. Answer (D) is incorrect because no inventory should be
included in the 2001 financial statements, and freight-in should be recorded in 2002.
xiv
.(c) Inventoriable costs include all costs necessary to prepare goods for sale. For a merchandising concern, these
include the purchase price of the goods, freight-in, insurance, warehousing, and any costs necessary to get the goods to
the point of sale. Abnormal freight and handling should be charged to expense of the period. Therefore, the normal costs
for inventory are $5,500 ($3,000 + $2,000 + $500) and the abnormal freight of $1,200 is charged to current expense of
the period. ·
xv
.$990 ($1,000 x 99%)
xvi
.$980 ($1,000 x 98%)
xvii
.P5,697.72 (P8,000 x 85% x 90% x 95% x 98%)
xviii
.P106,832.25 (P109,012.50 x 98%)
xix
.($1,800 - $200) x .98 = $1,568
xx
..98 (1/2 x 3,000) = 1,470
xxi
. [($4,000 - $275) x .98] + $350 = $4,000.50
xxii
.P11,200 (P12,000 – P800)
xxiii
.Answer (B) is correct. Revenue is recognized when it is realized or realizable and earned. Revenue is ordinarily earned
upon delivery. Given that 50% of the heating oil was delivered in 2000, 50% of the price was earned in 2000. Thus,
Acme should recognize $150,000 (50% x $300,000) of revenue from the sale.
Answer (A) is incorrect because $75,000 was the amount due on first delivery. Answer (C) is incorrect because
$225,000 was the amount due in 2000. Answer (D) is incorrect because $300,000 is the total price, but this amount has
not been earned because the last 50,000 gallons were not delivered in 2000.
xxiv
.$8,532 [($15,000 x 70% x 80% x 98%) + $300]
xxv
.P3,620,000 [(P5,000,000 x 80% x 90% x 95%) + P200,000]
xxvi
.The correct answer is (C). REQUIRED: The amount received as a remittance in full payment.
DISCUSSION: Inventory sold should always be invoiced net of trade discounts. Remittances paid during the cash
or purchase discount period should be net of these discounts. When goods are shipped FOB shipping point, they
become the purchaser’s inventory at the time of shipment. Thus, the purchaser is responsible for the payment of
delivery costs. As indicated below, the remittance received by Halle should amount to $2.944.
List price $5,000
30% trade discount (1,500)
$3,500
20% trade discount (700)
$2,800
2% cash discount (56)
$2,744
Delivery costs 200
$2,944
Answer (A) is incorrect because $2,744 excludes the delivery costs. Answer (B) is incorrect because $2,940
includes a 2% cash discount on the delivery costs. Answer (D) is incorrect because $3,140 includes a 2% discount
on the delivery costs and double counts the delivery costs.

TRANSACTION ANALYSIS
Effect on Asset, Liabilities & Equity
Cash sale
47. Stone Company sold inventory costing $700 for $850 on account. If Stone Company operates under the accrual basis,
what effect will this transaction have on the owners' equity side of the balance sheet?
a. none since the customer to whom the inventory was sold has not yet paid
b. none since sales and/or cost of goods sold are income statement accounts
c. decrease owners' equity by $700
d. increase owners' equity by $150
e. increase owners' equity by $850

PERPETUAL INVENTORY SYSTEM


Purchases & Payments
Questions 65 thru 70 are based on the following information.
Based on the following information,
1. $5,000 of merchandise inventory was ordered on April 2, 2007
2. $2,000 of this merchandise was received on April 5, 2007
3. On April 6, 2007, an invoice dated April 4, 2007, with terms of 2/10, net 30 for $2,150 which included a $150
prepaid freight cost, was received.
4. On April 10, 2007, $500 of the merchandise was returned to the seller.

68. The entry for April 10, 2007 would include? (D)
a. Debit to Merchandise Inventory $500 c. Credit to Merchandise Inventory $500
b. Debit to Purchases Returns $500 d. Credit to Accounts Payable $500
65. What would be recorded as purchases discount if the invoice is paid within the discount period? (M*)
a. $100 c. $43
b. $30 d. $33

66. What would be recorded as the cash payment if the invoice is paid within the discount period? (E*)
a. $1,470 c. $2,150
b. $1,520 d. $1,620

67. What would be recorded as net purchases amount after all of the transactions have been recorded? (D)
a. $2,000 c. $1,620
b. $2,150 d. $1,470

69. By what date does the invoice need to be paid in order to take the advantage of the discount? (D)
a. April 15, 2007 c. April 10, 2007
b. April 16, 2007 d. April 14, 2007

70. What would be the cash payment if the company decides to pay the invoice on April 30, 2007? (M)
a. $1,650 c. $2,150
b. $1,620 d. $2,000

COMPLETING THE ACCOUNTING CYCLE FOR A MERCHANDISING BUSINESS

TRIAL BALANCE
Total credits
83. Given the following complete list of balances, what will be the total credits in the trial balance, assuming
no errors exist in the accounts?
1. Retained Earnings $ 28,000
2. Merchandise Inventory 9,000
3. Accumulated Depreciation 5,000
4. Sales 42,000
5. Selling Expenses 11,000
6. Accounts Receivable 7,000
7. Cost of Goods Sold 22,000
8. Accounts Payable ?
9. Cash 5,000
10. Equipment 33,000
Note: The accounts payable records were damaged by a flood, and the company is not certain what the correct
balance should be.
a. $72,000
b. $69,000
c. $58,000
d. $87,000
e. Due to the damage of the accounts payable records, it is impossible to determine the amount of
the total credits on the trial balance.

INCOME STATEMENT
Sales Revenue
Gross Sales
Common size income statement
. A company has a 50% gross margin, general and administrative expenses of $50, interest expense of
$20, and net income of $10 for the year just ended. If the corporate tax rate is 50%, the level of sales revenue for
the year just ended was (E)
A. $90 C. $150
B. $135 D. $180 CIA
1194, 0596

. A company has a 40% gross margin, general and administrative expenses of $50, interest expense of $20, and net
income of $70 for the year just ended. If the corporate tax rate is 30%, the level of sales revenue for the year just ended
was (E)
a. $170 c. $350
b. $255 d. $425 CIA
1194

. A company has a 50% gross margin, general and administrative expenses of $50, interest expense of
$20, and net income of $10 for the year just ended. If the corporate tax rate is 50%, the level of sales revenue for
the year just ended was (E)
A. $90 C. $150
B. $135 D. $180 CIA
1194

Net sales
Sales – sales returns & allowances – sales discount
113. Herald Company had sales of $135,000, sales discounts of $2,000, and sales returns of $3,200. Herald
Company's net sales equals: (E)
A. $5,200. D. $135,000.
B. $129,800. E. $140,200.
C. $133,000.

27. Silver Co. sold merchandise to Bronze Co. on account, $23,000, terms 2/15, net 45. The cost of the
merchandise sold is $18,500. Silver Co. issued a credit memorandum for $2,500 for merchandise returned that
originally cost $1,900. The Bronze Co. paid the invoice within the discount period. What is amount of net sales
from the above transactions? (E)
a. $20,090 c. $3,490
b. $20,500 d. $23,000

47. Neptune Company's gross sales in 2004 were $3,930,000. Assuming sales returns and allowances were $74,000, sales
discounts were $35,000, and freight-out was $28,000, what were Neptune's net sales in 2004? (E)
a. $3,793,000 c. $3,856,000
b. $3,821,000 d. $3,930,000
S, S & S

27. Gross billings for merchandise sold by Pye Company to its customers last year amounted to $12,720,000; sales returns
and allowances were $270,000, sales discounts were $175,000, and freight-out was $140,000. Net sales last year for
Pye Company were (E)
a. $12,720,000. c. $12,275,000.
b. $12,450,000. d. $12,135,000.
K, W & W

130. Financial information is presented below:


Operating Expenses $ 45,000
Sales Returns and Allowances 13,000
Sales Discounts 6,000
Sales 160,000
Cost of Goods Sold 77,000
The amount of net sales on the income statement would be (E)
a. $154,000. c. $160,000.
b. $141,000. d. $166,000.

111. Thelman Company reported the following balances at June 30, 2008:
Sales $10,800
Sales Returns and Allowances 400
Sales Discounts 200
Cost of Goods Sold 5,000
Net sales for the month is (E)
a. $10,800. c. $10,200.
b. $10,400. d. $5,200.

Cash sales + credit sales – sales returns & allowances – sales discount
. Benson Company had cash sales of $94,275, credit sales of $83,450, sales returns and allowances of
$1,700, and sales discounts of $3,475. Benson's net sales for this period equal: (E)
A. $ 94,275. D. $176,025.
B. $172,550. E. $177,725.
C. $174,250.

Gross profit + (purchases – ending inventory)


30. Hernandez Company had a gross profit of $240,000, total purchases of $280,000, and an ending
inventory of $160,000 in its first year of operations as a retailer. Hernandez’s sales in its first year must have been
(E)
a. $360,000.c. $120,000.
b. $440,000. d. $400,000.
K, W & W

Cost of Goods Sold


Beginning Inventories
Sales ÷ (1+markup) + ending inventory – (purchases + freight-in)
. A retail enterprise maintains a markup of 25% based on cost. The enterprise has the following information for 2001:
Purchases of merchandise 690,000
Freight-in on purchases 25,000
Sales 900,000
Ending inventory 80,000
Beginning inventory was (M)
A. 40,000 C. 110,000
B. 85,000 D. 265,000
CIA 0597

(Ending invty + Cost of goods sold) – (purchases + freight – purchase returns)


. Aphrodite Corporation’s income statement showed $78,000 in cost of goods sold for the year ending
December 31, 2006. During that same year, merchandise inventory was purchased for $80,000, freight-in
amounted to $300, defective merchandise worth $500 was returned and the ending merchandise inventory
balance was $11,900. Based on this information, what must beginning inventory have been?
A. $13,700 C. $10,100
B. $10,400 D. $9,900

Cost of goods purchased


Sales x (1 – gross profit ratio) + increase in inventory
. At the beginning of 2007, Grand Hardware has an inventory of $300,000. Because sales growth was strong
during 2007, the owner wants to increase inventory on hand to $350,000 at December 31, 2007. If net sales for
2007 are expected to be $1,500,000, and the gross profit rate is expected to be 35%, compute the cost of the
merchandise the owner should expect to purchase during 2007. (M)
A. $ 925,000. C. $1,025,000.
B. $ 975,000. D. Some other amount.

. On July 1, the inventory of at Comfee Shoes was $50,000. Because of anticipated back-to-school sales, the
owner wants to have an inventory of $95,000 on hand at the beginning of August. Net sales during July are
expected to total $60,000, with a gross profit rate of 45%. During July, the company should purchase merchandise
costing:
A. $57,500. C. $78,000.
B. $128,000. D. Some other amount.

. At the beginning of 2007, England Dresses has an inventory of $140,000. However, management wants
to reduce the amount of inventory on hand to $80,000 at December 31. If net sales for 2007 are forecast at
$400,000 and the gross profit rate is expected to be 40%, compute the cost of the merchandise which
management should expect to purchase during 2007. (Hint: First compute the expected cost of goods sold.)
A. $240,000. C. $320,000.
B. $180,000. D. Some other amount.

Sales x (1 – gross profit ratio) – decrease in inventory


. The following information was obtained from Smith Co.:
Sales $275,000
Beginning inventory 30,000
Ending inventory 18,000
Smith's gross margin is 20%. What amount represents Smith purchases?
a. $202,000 c. $220,000
b. $208,000 d. $232,000
AICPA R05

. At the beginning of 2007, England Dresses has an inventory of $140,000. However, management wants to
reduce the amount of inventory on hand to $80,000 at December 31. If net sales for 2007 are forecast at $400,000
and the gross profit rate is expected to be 40%, compute the cost of the merchandise which management should
expect to purchase during 2007. (Hint: First compute the expected cost of goods sold.)
A. $240,000. C. $320,000.
B. $180,000. D. Some other amount.

Freight-in
COGAS – (beg. inventory + purchases – purchase returns & allowances – purchase discount)
89. From the following information, determine the amount of freight-in. (E)
Beginning Inventory ................................... $20,000
Purchases ............................................. 41,000
Purchase Returns and Allowances ....................... 3,000
Purchase Discounts .................................... 4,000
Freight-In ............................................ ?
Cost of Goods Available for Sale ...................... 55,000
Ending Inventory ...................................... ?
Cost of Goods Sold .................................... 22,000
A. $3,000 C. $2,000
B. $4,000 D. $1,000 S, S &
S

Net purchases
Purchases + freight-in – purchase discounts, returns & allowances
. An enterprise had the following account balances in the pre-closing trial balance:
Opening inventory 100,000
Closing inventory 150,000
Purchases 400,000
Transportation-in 6,000
Purchase discounts 40,000
Purchase allowances 15,000
Returned purchases 5,000
The enterprise had net purchases for the period of (E)
A. 340,000 C. 370,000
B. 346,000 D. 376,000
CIA 1195 IV-5 & 6

Cost of goods available for sale


Net sales x (1 -- gross profit ratio) + ending inventory
. The following information is available for Cooke Company for 2010:
Net sales $1,800,000
Freight-in 45,000
Purchase discounts 25,000
Ending inventory 120,000
The gross margin is 40% of net sales. What is the cost of goods available for sale?
A: $ 840,000 C: $1,200,000
B: $ 960,000 D: $1,220,000
Wiley 2011

Beg. inventory + (purchases + freight-in – purchase returns)


146. Baden Shoe Store has a beginning merchandise inventory of $30,000. During the period, purchases were
$140,000; purchase returns, $4,000; and freight-in $10,000. A physical count of inventory at the end of the period
revealed that $20,000 was still on hand. The cost of goods available for sale was (E*)
a. $164,000. c. $176,000.
b. $156,000. d. $184,000.

Beg. inventory + (purchases + freight- in - purchase returns & discount)


. Outback Furriers started 2003 with $94,000 of merchandise inventory on hand. During 2003, $400,000 in merchandise
was purchased on account with credit terms of 1/15 n/45. All discounts were taken. Purchases were all made f.o.b.
shipping point. Outback paid freight charges of $6,000. Merchandise with an invoice amount of $5,000 was returned for
credit. Cost of goods sold for the year was $380,000. Outback uses a perpetual inventory system. Assuming Outback
uses the gross method to record purchases, what is the cost of goods available for sale? (M)
A. $491,000. C. $489,000.
B. $495,000. D. $491,050.
S, S & T

. Burnit Bakeries started 2003 with $62,000 of merchandise on hand. During 2003, $280,000 in merchandise was
purchased on account with credit terms of 2/10 n/30. All discounts were taken. Purchases were all made f.o.b. shipping
point. Burnit paid freight charges of $9,000. Merchandise with an invoice amount of $4,000 was returned for credit. Cost
of goods sold for the year was $316,000. Burnit uses a perpetual inventory system. What is cost of goods available for
sale, assuming Burnit uses the gross method? (M)
A. $322,480. C. $336,000.
B. $341,480. D. $347,000.
S, S & T

Beg. inventory + (purchases + freight-in – purchase returns, allowances & discounts)


88. Following are the account balances from Jackson Company's income statement:
Inventory, January 1, 2005 ............................ $35,000
Purchases ............................................. 35,000
Purchase Returns and Allowances ....................... 2,000
Purchase Discounts .................................... 4,000
Freight-In ............................................ 5,000
Inventory, December 31, 2005 .......................... 10,000
Freight-Out ........................................... 6,000
Given this information, the cost of merchandise available for sale during 2005 is (E)
A. $65,000. C. $69,000.
B. $59,000. D. $61,000. S, S & S

93. The following information was obtained from the accounts of Cox Company:
Beginning Inventory .................................. $20,000
Purchases ............................................ 40,000
Purchase Returns and Allowances ...................... 2,000
Purchase Discounts ................................... 4,000
Freight-In ........................................... 5,000
Ending Inventory ..................................... 10,000
Freight-Out .......................................... 6,000
Given this information, the cost of goods available for sale is (E)
A. $65,000. C. $69,000.
B. $59,000. D. $61,000.
S, S & S

Ending Inventory
Cost of goods available for sale -- cost of sales
90. From the following information, determine the amount of ending inventory. (E)
Beginning Inventory ................................... $20,000
Purchases ............................................. 41,000
Purchase Returns and Allowances ....................... 3,000
Purchase Discounts .................................... 4,000
Freight-In ............................................ ?
Cost of Goods Available for Sale ...................... 55,000
Ending Inventory ...................................... ?
Cost of Goods Sold .................................... 22,000
A. $23,000 C. $33,000
B. $32,000 D. $22,000
S, S & S

(Beg invty + purchases) – (sales – gross profit)


. Chenelle Company reported the following information on December 31, 2006:
Sales $120,000 Beg. Inventory (Dec 1) $8,000
Purchases 90,000 Gross Margin 32,000
Based on the information provided, what was Chenelle’s ending inventory balance as of December 31, 2006?
A. $88,000 C. $10,000
B. $18,000 D. $8,000

(Beginning inventory + purchases + freight-in – discounts) - cost of sales


. Outback Furriers started 2003 with $94,000 of merchandise inventory on hand. During 2003, $400,000 in merchandise
was purchased on account with credit terms of 1/15 n/45. All discounts were taken. Purchases were all made f.o.b.
shipping point. Outback paid freight charges of $6,000. Merchandise with an invoice amount of $5,000 was returned for
credit. Cost of goods sold for the year was $380,000. Outback uses a perpetual inventory system. What is ending
inventory assuming Outback uses the gross method to record purchases? (E)
A. $110,990. C. $115,000.
B. $111,050. D. $118,800. S, S & T

. Burnit Bakeries started 2003 with $62,000 of merchandise on hand. During 2003, $280,000 in merchandise was
purchased on account with credit terms of 2/10 n/30. All discounts were taken. Purchases were all made f.o.b. shipping
point. Burnit paid freight charges of $9,000. Merchandise with an invoice amount of $4,000 was returned for credit. Cost
of goods sold for the year was $316,000. Burnit uses a perpetual inventory system. Assuming Burnit uses the gross
method to record purchases, ending inventory would be: (E)
A. $16,480. C. $31,000.
B. $25,400. D. $25,480.
S, S & T

(Beg inventory + purchases) – [(collection -- beg + ending A/R) x markup]


30. Brad Company prepared monthly income statements. A physical inventory is taken only at year-end,
hence month-end inventories must be estimated. All sales are made on account. The rate of markup on cost is
50%. The following information relates to the month of September:
Accounts receivable, September 1 P100,000
Accounts receivable, September 30 150,000
Collection of accounts receivable during September 250,000
Inventory, September 1 180,000
Purchases of inventory during September 160,000
The estimated cost of September 30 inventory is (M)
a. P120,000 c. P190,000
b. P140,000 d. P220,000
RPCPA 1095

. Finland Co. prepares monthly income statements. A physical inventory is taken only at year-end; hence, month-end
inventories must be estimated. All sales are made on account. The rate of markup on cost is 50%. The following
information related to the month of June:
Accounts receivable, June 1 $20,000
Accounts receivable, June 30 30,000
Collection of accounts receivable during June 50,000
Inventory, June 1 36,000
Purchase of inventory during June 32,000
The estimated cost of the June 30 inventory is (M)
a. $24,000 c. $38,000
b. $28,000 d. $44,000AICPA
1173 I-10

15. The YVC Corp. prepares monthly income statements. As a cost saving device, a physical inventory is taken only at
year-end, hence, month-end inventories must be estimated. All sales are made on account. The rate of markup on cost
is 50%.
Accounts receivable, March 1, 1980 60,000
Accounts receivable, March 31, 1980 90,000
Collection of accounts receivable 150,000
Inventory, March 1, 1980 P108,000
Purchase of inventory 96,000
The estimated cost of the March 31, 1980 inventory is (M)
a. P79,000 c. P88,000
b. P84,000 d. P93,000
RPCPA 0580

40. Jupiter Company prepares monthly income statements. A physical inventory is taken only at year-end;
hence, month-end inventories must be estimated. All sales are made on account. The rate of markup on cost is 50
percent. The following information relates to the month of May:
Accounts receivable, May 1 ............................ $20,000
Accounts receivable, May 31 ........................... 30,000
Collection of accounts receivable during May .......... 50,000
Inventory, May 1 ...................................... 36,000
Purchases of inventory during May ..................... 32,000
The estimated cost of the May 31 inventory is (M)
a. $24,000. c. $38,000.
b. $28,000. d. $44,000.
S, S & S

(Beginning inventory + purchases) – (sales - gross profit)


. The following amounts relate to an enterprise:
Beginning inventory 50,000
Purchases 170,000
Sales 180,000
Gross margin 60,000
The amount of ending inventory is (E)
A. 60,000 C. 120,000
B. 100,000 D. 160,000
CIA 0597 IV-11

58. Delta Merchandising, Inc., has provided the following information for the year just ended:
Beginning inventory 24,000
Purchases 80,000
Net sales $128,500
Gross margin 38,550
The ending inventory for the company at year end was: (E)
a. $65,450. c. $14,050.
b. $24,500. d. $9,950. G & N 9e

(Beginning inventory + purchases) – [sales x (1-- gross profit ratio)]


3. The merchandise inventory of CD Ltd. On January 1, 1988 was P450,000. During 1988, the company has recorded
sales of P1,500,000 and merchandise purchases of P1,100,000. The gross profit (margin) on these sales was 30%.
What is the merchandise inventory as at December 31, 1988? (M)
a. P500,000 c. P1,100,000
b. P650,000 d. Answer not
given. RPCPA 1089

7. From the information below for the period July 1 to September 30, compute the estimated inventory at
September 30 (M)
Sales, net, for the period 440,000
Gross profit (margin) on sales 35%
Inventory, July 1 P 45,000
Purchases, net, for the period 300,000
a. P95,000 c. P54,000
b. P59,000 d. None of these.
RPCPA 1077

(Beginning inventory + purchases + freight) – [sales x (1 -- gross profit ratio)]


. The following information is available for the Silver Company for the 3 months ended March 31, 2011.
Sales 4,800,000
Merchandise inventory, January 1, 2011 $ 900,000
Purchases 3,400,000
Freight-in 200,000
The gross margin recorded was 25% of sales. What should be the merchandise inventory at March 31, 2011? (M)
A: $ 700,000 C: $1,125,000
B: $ 900,000 D: $1,200,000
Wiley 2011

Purchases, Discount, ending inventory as % of purchases


22. The T Corporation made purchases of P114,000 during the year on terms of 2/10, n/30. The company
took advantage of the discount on 60% of the purchases. It paid for the remainder of the account after the
discount period had expired. The company uses the periodic inventory system. Half of the purchases are still in
the inventory at the end of the year.
The cost of ending inventory is (E)
A. P56,772 C. P56,316
B. P55,860 D. P67,032
RPCPA 0594

Cannot be determined
. Sam’s Company uses a periodic inventory system. On January 1, the beginning of the year, Sam’s had
an inventory balance of $5,000. During the year, Sam’s purchased $75,000 worth of merchandise and had sales
amounting to $125,000. On December 31, prior to adjusting and closing entries, what inventory balance should
Sam’s show?
A. Less than $5,000
B. $5,000
C. More than $5,000
D. Cannot be determined from the information provided

Shipping cost in ending inventory


. Seafood Trading Co. commenced operations during the year as a large importer and exporter of seafood.
The imports were all from one country overseas. The export sales were conducted as drop shipments and were
merely transshipped at Seattle. Seafood Trading reported the following data:
Purchases during the year $12.0 million
Shipping costs from overseas 1.5 million
Shipping costs to export customers 1.0 million
Inventory at year-end 3.0 million
What amount of shipping costs should be included in Seafood Trading’s year-end inventory valuation? (E)
A: $0 C: $375,000
B: $250,000 D: $625,000
AICPA R08

Cost of goods sold


Beg. inventory + purchases – ending inventory
. Given a periodic inventory system, beginning inventory of $30,000, ending inventory of $34,000, and
purchases of $450,000, what is the net debit to cost of goods sold? (E)
A. $450,000 C. $34,000
B. $30,000 D. $446,000

60. Gabel Inc. is a merchandising company. Last month the company's merchandise purchases totaled $63,000. The
company's beginning merchandise inventory was $13,000 and its ending merchandise inventory was $15,000. What
was the company's cost of goods sold for the month? (E)
a. $91,000 c. $65,000
b. $63,000 d. $61,000 G & N 9e

78. Gabriel Inc. is a merchandising company. Last month the company's merchandise purchases totaled
$70,000. The company's beginning merchandise inventory was $15,000 and its ending merchandise inventory
was $22,000. What was the company's cost of goods sold for the month? (E)
A. $63,000 C. $107,000
B. $77,000 D. $70,000
G & N 10e

77. Gabor Inc. is a merchandising company. Last month the company's merchandise purchases totaled
$89,000. The company's beginning merchandise inventory was $13,000 and its ending merchandise inventory
was $16,000. What was the company's cost of goods sold for the month? (E)
A. $89,000 C. $118,000
B. $86,000 D. $92,000
G & N 10e

7. A company, using the periodic inventory system, has merchandise inventory costing $140 on hand at the
beginning of the period. During the period, merchandise costing $400 is purchased. At year-end, merchandise
inventory costing $180 is on hand. The cost of merchandise sold for the year is (E)
a. $720 c. $360
b. $550 d. $140

139. At the beginning of September, 2008, RFI Company reported Merchandise Inventory of $4,000. During
the month, the company made purchases of $7,800. At September 31, 2008, a physical count of inventory
reported $3,200 on hand. Cost of goods sold for the month is (E)
a. $600. c. $8,600.
b. $7,800. d. $11,800.

76. Gable Inc. is a merchandising company. Last month the company's merchandise purchases totaled
$86,000. The company's beginning merchandise inventory was $15,000 and its ending merchandise inventory
was $11,000. What was the company's cost of goods sold for the month? (E)
A. $86,000 C. $82,000
B. $112,000 D. $90,000
G & N 10e

. Michael uses its periodic inventory system and the following information is available:
Sales $43,400
Inventory – Beginning 11,200
Inventory – Ending 9,800
Purchases 32,200
Calculate the cost of goods sold (E)
A. $ 9,800. C. $32,200.
B. $33,600. D. $43,400.

45. Given the following data, what is cost of goods sold? (E)
Sales revenue $845,000
Beginning inventory 110,000
Ending inventory 200,000
Purchases of inventory 705,000
a. $615,000 d. $905,000
b. $815,000 e. $735,000
c. $320,000

46. Given the following data, what is cost of goods sold?


Sales revenue $10,000
Beginning inventory 3,000
Ending inventory 7,000
Purchases of inventory 5,000
a. $12,000 d. $ 7,000
b. $ 9,000 e. $ 1,000
c. $ 8,000

Beg. inventory + purchases + freight-in – ending inventory


. Glen Company has the following data pertaining to the year ended December 31:
Purchases $450,000
Beginning inventory 170,000
Ending inventory 210,000
Freight-in 50,000
Freight-out 75,000
How much is the cost of goods sold for the year? (E)
a. $385,000 c. $485,000
b. $460,000 d. $535,000
AICPA 0586 I-44

. Glen Company has the following data pertaining to the year ended December 31, 2004:
Purchases 4,500,000
Beginning inventory 1,700,000
Ending inventory 2,100,000
Freight in 500,000
Freight out 750,000
How much is the cost of goods sold for 2004? (E)
A. 3,850,000 C. 4,850,000
B. 4,600,000 D. 5,400,000
AICPA

Beg. inventory + purchases – purchase discount – ending inventory


106. The following information is available from Preston Company's 2005 accounting records:
Purchases ............................................ $530,000
Purchase discounts ................................... 10,000
Beginning inventory .................................. 160,000
Ending inventory ..................................... 215,000
Freight-out .......................................... 40,000
Preston's 2005 cost of goods sold is (E)
A. $465,000. C. $505,000.
B. $475,000. D. $585,000. S, S & S

Beg. invty + (purchases + freight-in – purchase discount) - ending invty,


. The following information pertained to Azur Co. for the year:
Purchases $102,800
Purchase discounts 10,280
Freight-in 15,420
Freight-out 5,140
Beginning inventory 30,840
Ending inventory 20,560
What amount should Azur report as cost of goods sold for the year? (E)
A. $102,800 C. $123,360
B. $118,220 D. $128,500
AICPA R99

Beg. invty + purchases – purchase return – purchase discount – ending invty


. Aphrodite Corporation started the year with $16,000 in merchandise inventory. During the year,
purchases in the amount of $12,360 were made and $200 in damaged merchandise inventory was returned. The
company took advantage of $1,800 worth of purchase discounts during the year. If the ending inventory for
Aphrodite was $15,000, what amount of cost of goods sold was reflected on the income statement?
A. $9,360 C. $11,360
B. $10,360 D. $13,160

Beg. invty + (purchases + freight-in – purchase returns, allowances & discounts) – ending invty
. An enterprise had the following account balances in the pre-closing trial balance:
Opening inventory 100,000
Closing inventory 150,000
Purchases 400,000
Transportation-in 6,000
Purchase discounts 40,000
Purchase allowances 15,000
Returned purchases 5,000
If net purchases for the enterprise equal 500,000 for the period, cost of goods sold is (E)
A. 250,000 C. 550,000
B. 450,000 D. 750,000
CIA 1195 IV-5 & 6

91. The following information was obtained from the accounts of McKay Company:
Inventory, January 1 .................................. $30,000
Purchases ............................................. 45,000
Purchase Returns and Allowances ....................... 5,000
Purchase Discounts .................................... 4,000
Freight-In ............................................ 5,000
Inventory, December 31 ................................ 20,000
Freight-Out ........................................... 6,000
Given this information, the cost of goods sold during the year is (E)
A. $46,000. C. $51,000.
B. $41,000. D. $61,000. S, S & S

87. Following are the account balances from Fulton Company's income statement:
Inventory, January 1, 2005 ............................ $30,000
Purchases ............................................. 40,000
Purchase Returns and Allowances ....................... 5,000
Purchase Discounts .................................... 4,000
Freight-In ............................................ 5,000
Inventory, December 31, 2005 .......................... 15,000
Freight-Out ........................................... 6,000
Given this information, the cost of goods sold during 2005 is (E)
A. $51,000. C. $56,000.
B. $46,000. D. $66,000. S, S & S

Common-size income statement


9. Net income of Amity Co. for the month was P24,000. Selling expenses were equal to 15% of sales and
also 25% of the cost of sales. All other expenses were 13% of sales.
The cost of sales for the month was (M)
a. P200,000 c. P80,000
b. P120,000 d. Answer not
given RPCPA 0581

11. The following data pertain to the year 1987 of Molave Co.: Net income for the year was P28,000. Selling
expenses were equal to 18% of sales and also equal to 30% of the cost of sales. All other expenses were 12% of
sales.
The amount of cost of sales for the year 1987 of Molave Co. was (M)
a. P84,000 c. P168,000
b. P112,000 d. P251,000
RPCPA 0588

Cost of goods available for sale, ending inventory, cost of goods sold
Questions 1 thru 3 are based on the following information.
The following information was taken from Fruit-O Company accounting records:
Beginning Inventory $2,400 Purchases $22,000
Ending Inventory 1,600
Note: Fruit-O uses a periodic inventory system.

. Based on the information provided by Fruit-O Company, what is the cost of goods sold?
A. $22,000 C. $22,800
B. $20,400 D. $24,400

. Based on the information provided by Fruit-O Company, what is the cost of goods available for sale?
A. $24,400 C. $22,000
B. $22,800 D. $20,400

. Based on the information provided by Fruit-O Company, what amount of inventory will be shown on the
balance sheet at the end of the accounting period?
A. $24,400 C. $ 2,400
B. $22,800 D. $ 1,600

Gross profit & Gross Profit Percentage


Gross profit
Sales – cost of sales
. A company had sales of $695,000 and cost of goods sold of $278,000. Its gross margin equals (E)
A. $(417,000). D. $ 417,000.
B. $ 695,000. E. $ 973,000.
C. $ 278,000.

155. Cole Company has sales revenue of $39,000, cost of goods sold of $24,000 and operating expenses of
$9,000 for the year ended December 31. Cole's gross profit is (E)
a. $30,000. c. $6,000.
b. $15,000. d. $0.

126. Financial information is presented below:


Operating Expenses $ 45,000
Sales 150,000
Cost of Goods Sold 77,000
Gross profit would be (E)
a. $105,000. c. $73,000.
b. $28,000. d. $150,000.

. The following information is available:


Sales $2,250
Inventory – year-end 900
Purchases 1,350
Cost of goods sold 1,800
Calculate the gross profit (E)
A. $0. C. $450.
B. $900. D. $1,200

. Satellite Warehouse is a small retail business that specializes in the sale of top-of-the-line televisions.
This year, the store has begun to carry the Flat TV manufactured by Swan Co. Thus far this year, Satellite has
recorded the following transactions involving the Flat TV:
Jan. 5. Purchased 8 Flat TVs at a unit cost of $1,300
Jan. 18. Purchased 5 additional Flat TVs at $1,300 each
Feb. 12. Sold 9 Flat TVs to the Duke Hotel for $14,700
The gross profit on the Flat TVs as of February 12th is (E)
A. $11,700. C. $6,500.
B. $3,000. D. $14,700.

47. Given the following information, determine the gross profit.


Accounts Receivable $ 17,000
Administrative Expenses 24,000
Cost of Goods Sold 88,000
Depreciation Expense 5,000
Income Tax Expense 4,000
Inventory 26,000
Sales 242,000
Selling Expenses 36,000
Wage Expense 75,000
a. $ 11,000 d. $119,000
b. $ 15,000 e. $154,000
c. $ 39,000

Sales – (beg. inventory + purchases – ending inventory)


. Michael uses its periodic inventory system and the following information is available:
Sales $43,400
Inventory – Beginning 11,200
Inventory – Ending 9,800
Purchases 32,200
Calculate the gross profit.
A. $ 9,800 C. $32,200
B. $33,600 D. $43,400

Cost of goods sold ÷ (1 – gross profit ratio)


. If cost of goods sold is $240,000 and the gross profit rate is 40%, what is the gross profit? (E*)
A. $160,000 C. $240,000
B. $560,000 D. Some other amount.

. If costs of goods sold is $600,000 and its gross profit rate is 20%, what is the gross profit?
A. $150,000. C. $250,000.
B. $ 75,000. D. $100,000.

(Sales – sales returns, allowances & discount) – cost of goods sold


128. Financial information is presented below:
Operating Expenses $ 45,000
Sales Returns and Allowances 13,000
Sales Discounts 6,000
Sales 150,000
Cost of Goods Sold 67,000
Gross profit would be
a. $77,000. c. $70,000.
b. $64,000. d. $83,000.

131. Financial information is presented below:


Operating Expenses $ 45,000
Sales Returns and Allowances 13,000
Sales Discounts 6,000
Sales 160,000
Cost of Goods Sold 77,000
Gross profit would be
a. $77,000. c. $64,000.
b. $70,000. d. $83,000.

Sales – (beg. inventory + purchases – ending inventory)


. The following data appeared in the accounting records of a retail store for the year ended December 31,
2010:
Sales $150,000
Purchases 70,000
Inventories:
January 1 35,000
December 31 50,000
Sales commissions 5,000
How much was the gross margin?
A: $65,000 C: $90,000
B: $75,000 D: $95,000
Wiley 2011

. The following information appeared in the accounting records of a retail store for the year ended
December 31, 1988:
Sales $300,000
Purchases 140,000
Inventories
January 1 70,000
December 31 100,000
Sales commissions 10,000
The gross margin was:
a. $190,000 c. $160,000
b. $180,000 d. $150,000
AICPA 1189

Gross profit percentage


(Sales – cost of goods sold) ÷ Sales
. If Sam’s Company has net sales of $500,000 and its cost of goods sold is $350,000, the company’s gross
profit ratio is
A. 15% C. 70%
B. 30% D. 100%

127. Financial information is presented below:


Operating Expenses $ 45,000
Sales 150,000
Cost of Goods Sold 77,000
The gross profit rate would be
a. .700. c. .300.
b. .187. d. .487.

[(Sales – sales returns & allowances) – cost of goods sold] ÷ Sales


161. In the Clark Company, sales were $480,000, sales returns and allowances were $30,000, and cost of
goods sold was $288,000. The gross profit rate was
a. 64%. c. 40%.
b. 36%. d. 60%.

[(Sales – sales returns, allowances & discounts) – cost of goods sold] ÷ Net Sales
118. A company shows the following balances:
Sales $1,000,000
Sales Returns and Allowances 180,000
Sales Discounts 20,000
Cost of Goods Sold 560,000
What is the gross profit percentage? (M)
a. 56% c. 44%
b. 70% d. 30%

129. Financial information is presented below:


Operating Expenses $ 45,000
Sales Returns and Allowances 13,000
Sales Discounts 6,000
Sales 150,000
Cost of Goods Sold 67,000
The gross profit rate would be
a. .535. c. .511.
b. .489. d. .553.

132 Financial information is presented below:


Operating Expenses $ 45,000
Sales Returns and Allowances 13,000
Sales Discounts 6,000
Sales 160,000
Cost of Goods Sold 77,000
The gross profit rate would be
a. .454. c. .500.
b. .546. d. .538.
(Net sales – cost of sales)÷ net sales
133. If a company has sales of $420,000, net sales of $400,000, and cost of goods sold of $260,000, the gross
profit rate is (E)
a. 67%. c. 35%.
b. 65% d. 33%.

. J.C. Penny had net sales of $28,496 million, its cost of goods sold was $19,092 million, and its net income
was $997 million. Its gross margin ratio equals: (E)
A. 3.5%. D. 67%.
B. 5.2%. E. 149.3%.
C. 33%.

117. If a company has net sales of $500,000 and cost of goods sold of $350,000, the gross profit percentage is
(E)
a. 70%. c. 15%.
b. 30%. d. 100%.

9. A company has net sales of $56,700 and a cost of goods sold of $26,700. The company’s gross profit
percentage is approximately (E)
A. 52.9% C. 89.0%
B. 47.1% D. 189%

97. A company's net sales were $676,600, its cost of good sold was $236,810 and its net income was
$33,750. Its gross margin ratio equals: (E)
A. 5%. D. 65%.
B. 9.6%. E. 285.7%.
C. 35%.

. A company had net sales and cost of goods sold of $752,000 and $543,000, respectively. Its net income
was $17,530. The company's gross margin ratio equals: (E)
A. 18.9% D. 34.7%
B. 24.5% E. 35.2%
C. 27.8%

Gross profit ÷ net sales


. A company's gross profit was $83,750 and its net sales were $347,800. Its gross margin ratio equals: (E)
A. 4.2%. D. $ 83,750.
B. 24.1%. E. $264,050.
C. 75.9%.

Mark-up & Margin


31. A markup of 40% on cost is equivalent to what margin on selling price?
a. 29% c. 40%
b. 35% d. 60% K, W
&W

Sales, Cost of Sales, Gross Profit & Gross Profit Percentage


Net sales & cost of sales
134. Ingrid’s Fashions sold merchandise for $38,000 cash during the month of July. Returns that month totaled
$800. If the company’s gross profit rate is 40%, Ingrid’s will report monthly net sales revenue and cost of goods
sold of (E)
a. $38,000 and $22,800. c. $37,200 and $22,320.
b. $37,200 and $14,880. d. $38,000 and $22,320.

Cost of goods sold & ending inventory


. At the beginning of the year, Saratoga Dress Co. had an inventory of $300,000. During the year, the
company purchased merchandise costing $850,000. Net sales for the year totaled $1,200,000, and the gross profit
rate was 45%. The cost of goods sold and the ending inventory, respectively, were: (E*)
A. $1,150,000 and $660,000. C. $660,000 and $490,000.
B. $540,000 and $610,000. D. Some other amounts.

Cost of goods sold & gross profit rate


140. At the beginning of the year, Midtown Athletic had an inventory of $400,000. During the year, the company
purchased goods costing $1,600,000. If Midtown Athletic reported ending inventory of $600,000 and sales of
$2,000,000, the company’s cost of goods sold and gross profit rate must be (E)
a. $1,000,000 and 50%. c. $1,000,000 and 30%.
b. $1,400,000 and 30%. d. $1,400,000 and 70%.

. The gross sales for Jezzie Corporation in 2006 was $675,000. Gross profit from those sales amounted to
$270,000. Based on this information, the cost of goods sold and gross profit ratio for Jezzie Corporation is
A. $405,000, 60% C. $270,000, 60%
B. $405,000, 40% D. $270,000, 40%

Questions 1 & 2 are based on the following information.


Boyd Company recorded the following transactions during 2006: Note: Boyd uses a perpetual inventory system.
1. Started the period with $2,000 worth of merchandise inventory
2. Purchased $4,000 worth of merchandise on account with the terms 3/10, n/45
3. Paid $225 in shipping costs for goods purchased which were shipped FOB destination
4. Returned $400 worth of damaged goods
5. Paid for merchandise within the discount period and after the return of the merchandise
6. Had $8,000 in sales for the period
7. Ended the year with $492 worth of merchandise

. Based on the record of transactions provided by Boyd Company, what was cost of goods sold for this
period?
A. $4,988 C. $5,108
B. $5,000 D. $5,508

. Based on the information provided by Boyd Company, its gross profit ratio (rounded) is:
A. 31% C. 38%
B. 36% D. 63%

Net sales, shipping costs


Questions 1 & 2 are based on the following information.
Boyd Company recorded the following transactions during 2006:
1. On January 1, Boyd sold $8,000 worth of merchandise to a customer on account with the terms 2/10,
n/30.
2. Shipping costs for the sale amounted to $900 and the merchandise was shipped FOB shipping point.
3. On January 5, the customer returned $400 worth of damaged goods; the cost of these goods was $160.
4. On January 10, the customer paid for the goods.

. Based on the transactions provided by Boyd Company, what are the net sales for this period?
A. $7,440 C. $8,000
B. $7,448 D. $8,348

. Based on the information provided by Boyd Company, how would the shipping cost be recorded?
A. As an addition to merchandise inventory
B. As a product cost
C. As a selling expense
D. It wouldn’t be recorded by Boyd at all.

Gross margin & expenses


. A company had net sales and cost of goods sold of $752,000 and $543,000, respectively. Its net income
was $17,530. The company's gross margin and expenses are ________ and ____________, respectively. (D)
A. $209,000; $191,470 D. $227,000; $525,470
B. $191,470; $209,000 E. $734,000; $191,470
C. $525,470; $227,000

Gross profit & purchases


. Chenelle Company reported the following information on December 31, 2006:
Sales $100,000 Beg. Inventory (Dec. 1) $8,000
Ending Inventory 6,000 Cost of Goods Sold 60,000
Based on the information provided, what was Chenelle’s gross margin and what was the total amount of
merchandise that was purchased for the year?
A B C D
Gross margin $40,000 $34,000 $40,000 $34,000
Purchases $68,000 $46,000 $58,000 $40,000

Gross profit, gross profit ratio, profit margin


Questions 1 thru 3 are based on the following information.
Selected account information from Aphrodite Corporation is presented below:
Cost of Goods Sold $250,000 Net Sales $500,000
Merchandise Inventory 13,000 Accounts Receivable 6,000
Selling Expenses 115,000 Administrative Expenses 90,000

. Based on the information for Aphrodite Corporation presented above, the gross profit amount shown on
the company’s income statement would be:
A. $500,000 C. $135,000
B. $250,000 D. $45,000

. Based on the information for Aphrodite Corporation presented above, the gross profit ratio is
A. 100% C. 27%
B. 50% D. 9%

. Based on the information for Aphrodite Corporation presented above, the profit margin ratio is:
A. 50% C. 9%
B. 27% D. 6%
Net sales, gross profit, net income
Questions 1 thru 3 are based on the following information:
Selected account information from Aphrodite Corporation is presented below:
Cost of Goods Sold $77,000 Sales $150,000
Sales Returns & Allowances 13,000 Sales Discount 6,000
Selling Expenses 15,000 Administrative Expenses 30,000

. The amount of net sales shown on the company’s income statement would be
A. $131,000 C. $144,000
B. $137,000 D. $169,000

. The gross profit amount shown on the company’s income statement would be
A. $54,000 C. $73,000
B. $61,000 D. $76,000

. The amount of net income that would be shown on the company’s income statement is
A. $54,000 C. $28,000
B. $47,000 D. $9,000

Gross profit, income from operations, net income


Use the following information for questions 123–125.
During 2008, Salon Enterprises generated revenues of $60,000. The company’s expenses were as follows: cost of
goods sold of $30,000, operating expenses of $12,000 and a loss on the sale of equipment of $2,000.

123. Salon’s gross profit is


a. $60,000. c. $18,000.
b. $30,000. d. $16,000.

124. Salon’s income from operations is


a. $60,000. c. $18,000.
b. $30,000. d. $12,000.

125. Salon’s net income is


a. $60,000. c. $18,000.
b. $30,000. d. $16,000.

Net sales, COGAS, COGS, gross profit


Questions 22 thru 25 are based on the following information.
Using the following information
Purchases $28,000 Purchases discounts $800
Merchandise inventory April 1 6,500 Merchandise inventory April 30 7,800
Sales returns and allowances 750 Sales 57,000
Purchases returns and allowances 1,000 Transportation In 880

24. What is the amount of net sales? (M)


a. 25,780 c. 57,750
b. 57,000 d. 56,250
25. What is the amount of merchandise available for sale? (D)
a. 33,580 c. 25,780
b. 30,470 d. 34,500

22. What is the amount of cost of merchandise sold? (D)


a. 25,780 c. 31,220
b. 23,270 d. 24,020

23. What is the amount of gross profit? (D)


a. 31,970 c. 25,780
b. 30,470 d. 56,250

Gross profit, operating income, non-operating income, net income


Use the following information for questions 135–138.
During August, 2008, Sal’s Supply Store generated revenues of $30,000. The company’s expenses were as
follows: cost of goods sold of $12,000 and operating expenses of $2,000. The company also had rent revenue of
$500 and a gain on the sale of a delivery truck of $1,000.

135. Sal’s gross profit for August, 2008 is


a. $30,000. c. $18,000.
b. $19,000. d. $16,000.

136. Sal’s nonoperating income (loss) for the month of August, 2008 is
a. $0. c. $1,000.
b. $500. d. $1,500.

137. Sal’s operating income for the month of August, 2008 is


a. $30,000. c. $18,500.
b. $19,500. d. $16,000.

138. Sal’s net income for August, 2008 is


a. $18,000. c. $16,500.
b. $17,500. d. $16,000.

Cost of goods sold, gross profit, net income, profit margin


Questions 1 thru 4 are based on the following information.
The following information has been taken from Aphrodite’s accounting records:
Beginning Inventory $23,000 Sales $84,000
Ending Inventory 9,000 Sales Discount 1,700
Purchases 52,000 Freight-in 1,300
Sales Returns 200 Sales Allowance 700
Purchase Returns 200 Purchase Allowance 500
Selling & Admin. Expenses 3,000 Purchase Discounts 700

1. Based on the account detail provided by Aphrodite, and assuming that the company uses a periodic
inventory system, the company’s cost of goods sold would be
A. $64,600 C. $65,900
B. $64,700 D. $66,000
2. Based on the account detail provided by Aphrodite, its gross profit ratio (rounded to two decimal places) is
A. 19% C. 21%
B. 20% D. 22%

3. Based on the account detail provided by Aphrodite, what was the company’s net income (loss)?
A. $12,500 C. $18,500
B. $15,500 D. $23,200

4. Based on the account detail provided by Aphrodite, what was the company’s profit margin ratio
(rounded)?
A. 15% C. 19%
B. 18% D. 28%

INCOME STATEMENT
Net Sales Revenue
Sales, sales returns & allowances, sales discounts given
113.Herald Company had sales of $135,000, sales discounts of $2,000, and sales returns of $3,200. Herald Company's net
sales equals: (M)
A) $5,200. D) $135,000.
B) $129,800. E) $140,200.
C) $133,000.

111.Thelman Company reported the following balances at June 30, 2008:


Sales $10,800
Sales Returns and Allowances 400
Sales Discounts 200
Cost of Goods Sold 5,000
Net sales for the month is
a. $10,800. c. $10,200.
b. $10,400. d. $5,200.

130.Financial information is presented below:


Operating Expenses $ 45,000
Sales Returns and Allowances 13,000
Sales Discounts 6,000
Sales 160,000
Cost of Goods Sold 77,000
The amount of net sales on the income statement would be
a. $154,000. c. $160,000.
b. $141,000. d. $166,000.

Cash sales, credit sales, sales returns & allowances, sales discounts given
. Benson Company had cash sales of $94,275, credit sales of $83,450, sales returns and allowances of $1,700, and sales
discounts of $3,475. Benson's net sales for this period equal: (M)
A) $ 94,275. D) $176,025.
B) $172,550. E) $177,725.
C) $174,250.

Net Purchases
Purchases + freight in – purchase returns & allowances, -- purchase discount
6. A company using the periodic inventory system has the following account balances: Merchandise Inventory at the
beginning of the year, $4,000; Transportation-In, $450; Purchases, $12,000; Purchases Returns and Allowances,
$2,300; Purchases Discounts, $220. The cost of merchandise purchased is equal to (E)
a. $13,930 c. $9,489
b. $9,930 d. $14520

145.West Company has the following account balances:


Purchases $48,000
Sales Returns and Allowances 6,400
Purchase Discounts 4,000
Freight-in 3,000
Delivery Expense 4,000
The cost of goods purchased for the period is (E)
a. $52,000. c. $51,000.
b. $47,000. d. $44,600.

Increase in inventory, cost of goods sold given


. Dave's Duds reported cost of goods sold of $2,000,000 this year. The inventory account increased by $200,000 during
the year to an ending balance of $400,000. What was the cost of merchandise that Dave purchased during the year?
A) $1,600,000. C) $2,200,000.
B) $1,800,000. D) $2,400,000.

Decrease in inventory, cost of goods sold given


141.During the year, Darla’s Pet Shop’s merchandise inventory decreased by $20,000. If the company’s cost of goods sold
for the year was $300,000, purchases must have been
a. $320,000. c. $260,000.
b. $280,000. d. Unable to determine.

. During the year 2008, the inventory of Barbara's Gift Shop decreased by $40,000. If the income statement for the year
2008 reported cost of goods sold of $300,000, purchases during the year must have amounted to:
A. $350,000. C. $260,000.
B. $270,000. D. Some other amount.

Increase in inventory, net sales, gross profit rate given


. At the beginning of 2007, Grand Hardware has an inventory of $300,000. Because sales growth was strong during 2007,
the owner wants to increase inventory on hand to $350,000 at December 31, 2007. If net sales for 2007 are expected to
be $1,500,000, and the gross profit rate is expected to be 35%, compute the cost of the merchandise the owner should
expect to purchase during 2007.
A. $ 925,000. C. $1,025,000.
B. $ 975,000. D. Some other amount.

. On July 1, the inventory of at Comfee Shoes was $50,000. Because of anticipated back-to-school sales, the owner
wants to have an inventory of $95,000 on hand at the beginning of August. Net sales during July are expected to total
$60,000, with a gross profit rate of 45%. During July, the company should purchase merchandise costing:
A. $57,500. C. $78,000.
B. $128,000. D. Some other amount.

Decrease in inventory, net sales, gross profit rate given


. At the beginning of 2007, England Dresses has an inventory of $140,000. However, management wants to reduce the
amount of inventory on hand to $80,000 at December 31. If net sales for 2007 are forecast at $400,000 and the gross
profit rate is expected to be 40%, compute the cost of the merchandise which management should expect to purchase
during 2007. (Hint: First compute the expected cost of goods sold.)
A. $240,000. C. $320,000.
B. $180,000. D. Some other amount.

Cost of goods available for sale


146.Baden Shoe Store has a beginning merchandise inventory of $30,000. During the period, purchases were $140,000;
purchase returns, $4,000; and freight-in $10,000. A physical count of inventory at the end of the period revealed that
$20,000 was still on hand. The cost of goods available for sale was
a. $164,000. c. $176,000.
b. $156,000. d. $184,000.

. The following information applies to the Income Statement of Addison Company.


Gross sales $1,000,000
Net sales 900,000
Freight-in 10,000
Ending inventory 200,000
Gross profit margin 40%
Addison’s cost of goods available for sale is
a. $550,000. c. $740,000.
b. $560,000. d. $800,000.

Cost of goods sold


Sales, gross profit given
. A company had sales of $375,000 and its gross profit was $157,500. Its cost of goods sold equals: (M)
A) $(217,000). D) $ 217,500.
B) $ 375,000. E) $ 532,500.
C) $ 157,500.

78. Brooklyn’s Best Bar reports net sales of $750,000, gross profit of $325,000, and net income of $45,000. The company's
cost of goods sold is: (E)
A. $380,000. C. $470,000.
B. $425,000. D. Some other amount.

. Walnut Creek Supplies reports net sales of $3,000,000, net income of $300,000, and gross profit of $800,000. The
company's cost of goods sold is: (E)
A. $1,700,000. C. $500,000.
B. $1,900,000. D. Some other amount.

Beginning inventory, purchases, ending inventory given


7. A company, using the periodic inventory system, has merchandise inventory costing $140 on hand at the beginning of
the period. During the period, merchandise costing $400 is purchased. At year-end, merchandise inventory costing $180
is on hand. The cost of merchandise sold for the year is (E)
a. $720 c. $360
b. $550 d. $140

139.At the beginning of September, 2008, RFI Company reported Merchandise Inventory of $4,000. During the month, the
company made purchases of $7,800. At September 31, 2008, a physical count of inventory reported $3,200 on hand.
Cost of goods sold for the month is (E)
a. $600. c. $8,600.
b. $7,800. d. $11,800.

. Michael uses its periodic inventory system and the following information is available:
Sales $43,400
Inventory – Beginning 11,200
Inventory – Ending 9,800
Purchases 32,300
Calculate the cost of goods sold:
A. $ 9,800. C. $32,200.
B. $33,600. D. $43,400.

Gross profit
Sales, cost of sales given
155.Cole Company has sales revenue of $39,000, cost of goods sold of $24,000 and operating expenses of $9,000 for the
year ended December 31. Cole's gross profit is
a. $30,000. c. $6,000.
b. $15,000. d. $0.

. A company had sales of $695,000 and cost of goods sold of $278,000. Its gross margin equals:
A) $(417,000). D) $ 417,000.
B) $ 695,000. E) $ 973,000.
C) $ 278,000.

126.Financial information is presented below:


Operating Expenses $ 45,000
Sales 150,000
Cost of Goods Sold 77,000
Gross profit would be
a. $105,000. c. $73,000.
b. $28,000. d. $150,000.

. The following information is available:


Sales $2,250
Inventory at year-end 900
Purchases 1,350
Cost of goods sold 1,800
Calculate the gross profit:
A. $0. C. $450.
B. $900. D. $1,200

Sales, sales discount, cost of sales, freight-out given


. Gibbson Company sold inventory that cost $4,000 to Garrison Company for $6,000. The inventory was sold under the
terms 1/10, n/30. The receivable was collected within the discount period. The goods were delivered FOB destination.
Freight costs of $200 were paid in cash. Based on this information alone (and assuming the company uses a perpetual
inventory system), what would be the amount of gross margin that would show up on Gibbson’s income statement? (M)
A. $1,940 C. $1,800
B. $1,740 D. $2,000

Sales, sales returns & allowances, sales discounts, cost of sales given
128.Financial information is presented below:
Operating Expenses $ 45,000
Sales Returns and Allowances 13,000
Sales Discounts 6,000
Sales 150,000
Cost of Goods Sold 67,000
Gross profit would be
a. $77,000. c. $70,000.
b. $64,000. d. $83,000.
131.Financial information is presented below:
Operating Expenses $ 45,000
Sales Returns and Allowances 13,000
Sales Discounts 6,000
Sales 160,000
Cost of Goods Sold 77,000
Gross profit would be
a. $77,000. c. $64,000.
b. $70,000. d. $83,000.

Sales, beginning inventory, purchases, ending inventory given


. Michael uses its periodic inventory system and the following information is available:
Sales $43,400
Inventory – Beginning 11,200
Inventory – Ending 9,800
Purchases 32,300
Calculate the gross profit.
A. $ 9,800 C. $32,200
B. $33,600 D. $43,400

Cost of sales, gross profit rate given


. If cost of goods sold is $240,000 and the gross profit rate is 40%, what is the gross profit?
A. $160,000 C. $240,000
B. $560,000 D. Some other amount.

. If costs of goods sold is $600,000 and its gross profit rate is 20%, what is the gross profit?
A. $150,000. C. $250,000.
B. $ 75,000. D. $100,000.

Gross profit percentage


Gross profit, net sales given
. A company's gross profit was $83,750 and its net sales were $347,800. Its gross margin ratio equals: (E)
A) 4.2%. D) $ 83,750.
B) 24.1%. E) $264,050.
C) 75.9%.

Net sales, cost of sales given


117.If a company has net sales of $500,000 and cost of goods sold of $350,000, the gross profit percentage is
a. 70%. c. 15%.
b. 30%. d. 100%.

9. A company has net sales of $56,700 and a cost of goods sold of $26,700. The company’s gross profit percentage is
approximately:
A. 52.9% C. 89.0%
B. 47.1% D. 189%

97. A company's net sales were $676,600, its cost of good sold was $236,810 and its net income was $33,750. Its gross
margin ratio equals: (D)
A) 5%. D) 65%.
B) 9.6%. E) 285.7%.
C) 35%.

. A company had net sales and cost of goods sold of $752,000 and $543,000, respectively. Its net income was $17,530.
The company's gross margin ratio equals: (D)
A. 18.9% D. 34.7%
B. 24.5% E. 35.2%
C. 27.8%

. J.C. Penny had net sales of $28,496 million, its cost of goods sold was $19,092 million, and its net income was $997
million. Its gross margin ratio equals: (D)
A) 3.5%. D) 67%.
B) 5.2%. E) 149.3%.
C) 33%.

127.Financial information is presented below:


Operating Expenses $ 45,000
Sales 150,000
Cost of Goods Sold 77,000
The gross profit rate would be
a. .700. c. .300.
b. .187. d. .487.

Sales, net sales, cost of sales given


133.If a company has sales of $420,000, net sales of $400,000, and cost of goods sold of $260,000, the gross profit rate is
a. 67%. c. 35%.
b. 65% d. 33%.

Sales, sales returns & allowances, cost of sales given


161.In the Clark Company, sales were $480,000, sales returns and allowances were $30,000, and cost of goods sold was
$288,000. The gross profit rate was
a. 64%. c. 40%.
b. 36%. d. 60%.

Sales, sales returns & allowances, sales discounts, cost of sales given
118.A company shows the following balances:
Sales $1,000,000
Sales Returns and Allowances 180,000
Sales Discounts 20,000
Cost of Goods Sold 560,000
What is the gross profit percentage?
a. 56% c. 44%
b. 70% d. 30%

129.Financial information is presented below:


Operating Expenses $ 45,000
Sales Returns and Allowances 13,000
Sales Discounts 6,000
Sales 150,000
Cost of Goods Sold 67,000
The gross profit rate would be
a. .535. c. .511.
b. .489. d. .553.

132.Financial information is presented below:


Operating Expenses $ 45,000
Sales Returns and Allowances 13,000
Sales Discounts 6,000
Sales 160,000
Cost of Goods Sold 77,000
The gross profit rate would be
a. .454. c. .500.
b. .546. d. .538.

Net sales & cost of sales


134.Ingrid’s Fashions sold merchandise for $38,000 cash during the month of July. Returns that month totaled $800. If the
company’s gross profit rate is 40%, Ingrid’s will report monthly net sales revenue and cost of goods sold of
a. $38,000 and $22,800. c. $37,200 and $22,320.
b. $37,200 and $14,880. d. $38,000 and $22,320.

Cost of Goods Sold & Ending Inventory


. At the beginning of the year, Saratoga Dress Co. had an inventory of $300,000. During the year, the company
purchased merchandise costing $850,000. Net sales for the year totaled $1,200,000, and the gross profit rate was 45%.
The cost of goods sold and the ending inventory, respectively, were: (M)
A. $1,150,000 and $660,000. C. $660,000 and $490,000.
B. $540,000 and $610,000. D. Some other amounts.

Cost of goods sold & gross profit rate


140.At the beginning of the year, Midtown Athletic had an inventory of $400,000. During the year, the company purchased
goods costing $1,600,000. If Midtown Athletic reported ending inventory of $600,000 and sales of $2,000,000, the
company’s cost of goods sold and gross profit rate must be (M)
a. $1,000,000 and 50%. c. $1,000,000 and 30%.
b. $1,400,000 and 30%. d. $1,400,000 and 70%.

Gross margin ratio & expenses


. A company had net sales and cost of goods sold of $752,000 and $543,000, respectively. Its net income was $17,530.
The company's gross margin ratio and expenses are ________ and ____________, respectively. (D)
A. $209,000; $191,470 D. $227,000; $525,470
B. $191,470; $209,000 E. $734,000; $191,470
C. $525,470; $227,000

Net Income
27. A firm has the following accounts and financial data for 2002:
Sales revenue $3,060 Cost of goods sold $1,800
Accounts receivable 500 Preferred stock dividends 18
Interest expense 126 Tax rate 40%
Total operating expenses 600 Number of common shares
Accounts payable 240 outstanding 1,000
The firm's net profits after taxes for 2002 is ______.
A. -$206 C. $320
B. $213 D. $206 Gitman

Comprehensive
Questions 22 thru 25 are based on the following information.
Using the following information
Purchases $28,000 Purchases discounts $800
Merchandise inventory April 1 6,500 Merchandise inventory April 30 7,800
Sales returns and allowances 750 Sales 57,000
Purchases returns and allowances 1,000 Transportation In 880
24. What is the amount of net sales? (M)
a. 25,780 c. 57,750
b. 57,000 d. 56,250

25. What is the amount of merchandise available for sale? (D)


a. 33,580 c. 25,780
b. 30,470 d. 34,500

22. What is the amount of cost of merchandise sold? (D)


a. 25,780 c. 31,220
b. 23,270 d. 24,020

23. What is the amount of gross profit? (D)


a. 31,970 c. 25,780
b. 30,470 d. 56,250

Use the following information for questions 123–125.


During 2008, Salon Enterprises generated revenues of $60,000. The company’s expenses were as follows: cost of goods
sold of $30,000, operating expenses of $12,000 and a loss on the sale of equipment of $2,000.

123.Salon’s gross profit is


a. $60,000. c. $18,000.
b. $30,000. d. $16,000.

124.Salon’s income from operations is


a. $60,000. c. $18,000.
b. $30,000. d. $12,000.

125.Salon’s net income is


a. $60,000. c. $18,000.
b. $30,000. d. $16,000.

Use the following information for questions 135–138.


During August, 2008, Sal’s Supply Store generated revenues of $30,000. The company’s expenses were as follows: cost of
goods sold of $12,000 and operating expenses of $2,000. The company also had rent revenue of $500 and a gain on the
sale of a delivery truck of $1,000.

135.Sal’s gross profit for August, 2008 is


a. $30,000. c. $18,000.
b. $19,000. d. $16,000.

136.Sal’s nonoperating income (loss) for the month of August, 2008 is


a. $0. c. $1,000.
b. $500. d. $1,500.

137.Sal’s operating income for the month of August, 2008 is


a. $30,000. c. $18,500.
b. $19,500. d. $16,000.

138.Sal’s net income for August, 2008 is


a. $18,000. c. $16,500.
b. $17,500. d. $16,000.
Use the following to answer questions 91-94:
Satellite Warehouse is a small retail business that specializes in the sale of top-of-the-line televisions. This year, the store
has begun to carry the Flat TV manufactured by Swan Co. Thus far this year, Satellite has recorded the following
transactions involving the Flat TV:
Jan. 5. Purchased 8 Flat TVs at a unit cost of $1,300
Jan. 18. Purchased 5 additional Flat TVs at $1,300 each
Feb. 12. Sold 9 Flat TVs to the Duke Hotel for $14,700

. If Satellite uses a perpetual inventory system, the journal entry to record the purchase on January 18th would include
which of the following?
A. A debit to the Purchases account for $6,500.
B. A debit to the Cost of Goods Sold for $6,500
C. A credit to Inventory for $6,500.
D. A debit to Inventory for $6,500.

. The gross profit on the Flat TVs as of February 12th is:


A. $11,700. C. $6,500.
B. $3,000. D. $14,700.

93. If Satellite uses a perpetual inventory system, the journal entry to record the sale on February 12th would include all of
the following except:
A. A debit to the Cost of Goods Sold for $11,700.
B. A credit to Sales Revenue for $14,700.
C. A credit to Purchases for $11,700.
D. A credit to Inventory for $11,700.

94. Satellite maintains a subsidiary ledger account for each type of TV carried in the store. An examination of the account
for the Flat TV model at the end of February would show:
A. 4 units on hand with a total value of $1,300.
B. 4 units on hand with a total value of $5,200.
C. 13 units on hand with a total value of $16,900.
D. The amount that Satellite owes to Swan.

ACCOUNT ANALYSIS
Cash
72. Given the following transactions, what is the balance in the cash account?
1. The owner started the company by investing $8,900 cash.
2. The company paid $3,000 for six months' rent in advance.
3. The company acquired $2,400 in merchandise inventory with two-thirds of the purchase on account.
4. The company sold merchandise inventory costing $1,500 for $3,100 on account.
a. $3,600 debit balance d. $8,100 debit balance
b. $5,100 debit balance e. $8,100 credit balance
c. $5,100 credit balance

Merchandise inventory
Change in inventory balances
69. Zach’s Market recorded the following events involving a recent purchase of merchandise:
 Received goods for $50,000, terms 2/10, n/30.
 Returned $1,000 of the shipment for credit.
 Paid $250 freight on the shipment.
 Paid the invoice within the discount period.
As a result of these events, the company’s merchandise inventory
a. increased by $48,020. c. increased by $48,265.
b. increased by $49,250. d. increased by $48,270.

70. Jake’s Market recorded the following events involving a recent purchase of merchandise:
 Received goods for $20,000, terms 2/10, n/30.
 Returned $400 of the shipment for credit.
 Paid $100 freight on the shipment.
 Paid the invoice within the discount period.
As a result of these events, the company’s merchandise inventory
a. increased by $19,208. c. increased by $19,306.
b. increased by $19,700. d. increased by $19,308.

Sales Revenue, Receivables, Collection


Total Sales
Collections, Increase in receivables
94. Century Company's records were partially destroyed in a flood. The company does not know what sales have been for
the year, but it does know all sales were on account. Also, the beginning accounts receivable balance was $37,000, and
its accounts receivable balance at the time of the flood was $45,000. From the beginning of the year until the flood, cash
collections from credit customers was $172,000. Given this information, what are Century Company's sales for the year
until the flood? (E)
a. $180,000 d. $209,000
b. $172,000 e. $254,000
c. $164,000

Collection + decrease in unearned income


. An enterprise had cash receipts from sales of 175,000 during 2001, of which 30,000 was unearned at the end of 2001.
At the end of 2000, the company had 40,000 of unearned revenue, all of which was earned in 2001. The company's
sales revenue for 2001 would be (E)
A. 145,000 C. 175,000
B. 165,000 D. 185,000 CIA 0596 IV-4

Collection + increase in unearned income


. An enterprise had cash receipts from sales of 175,000 during 2001. The company had unearned revenue of 30,000 at
the end of 2000, and 40,000 at the end of 2001.
The company's sales revenue for 2001 would be (E)
A. 145,000 C. 175,000
B. 165,000 D. 185,000 HGT

Cash sales, collection from accounts, increase in receivables, write-off given


13. The total accounts receivable of the ABA Corporation are as follows: on January 1, 19x3 – P7,600; on January 31, 19x3
– P9,400. During the month P1,400 was received from cash sales, P18,000 was collected on accounts, accounts
receivable of P1,200 were written off as uncollectible, and allowance on sales of P500 were made. The total sales in
January is
a. P20,800 c. P20,400
b. P22,900 d. P18,500 RPCPA 0584

Cash sales, collection from accounts, decrease in receivables, cash sales,, write-off given
20. The following are some of the records of Constant Sales Company in the year 1986:
Accounts receivable, January 1, 1986 P260,000
Accounts receivable, December 31, 1986 154,000
Accounts receivable written off 6,000
Cash sales 270,000
Cash received on accounts receivable 450,000
The Gross Sales in 1986 is (E)
a. P604,000 c. P610,000
b. P620,000 d. P430,000 RPCPA 0587

19. The records of Super Co. for the year 1984 show the following:
Accounts receivable, Jan. 1 P105,000
Accounts receivable, Dec. 31 78,000
Accounts receivable written-off as uncollectible 2,000
Cash sales 150,000
Cash received on Accounts receivable 85,000
The gross sales for 1984 was (E)
a. P210,000 c. P228,000
b. P255,000 d. Answer not given. RPCPA 1086

14. For the month of December 1979, the records of JV Corp. show the following information:
Accounts receivable, Dec. 1, 1979 P328,600
Accounts receivable, Dec. 31, 1979 312,100
Accounts receivable written off as uncollectible 4,500
Cash sales 125,800
Cash received on accounts receivable 141,300
The company uses the direct write-off method in accounting for uncollectible accounts receivable.
Gross sales for December 1979 were (E)
a. P308,600 c. P316,500
b. P312,800 d. P255,100 RPCPA 0580

29. For the month of December, the records of Balin Corporation show the following information:
Accounts Receivable, December 1 ....................... 160,000
Accounts Receivable, December 31 ...................... 148,000
Cash sales ............................................ 60,000
Cash received on accounts receivable .................. $ 70,000
Accounts Receivable written off as uncollectible ...... 2,000
The corporation uses the direct write-off method in accounting for uncollectible accounts receivable. What are the gross
sales for the month of December? (E)
a. $118,000 c. $130,000
b. $120,000 d. $144,000 S, S & S

18. The accounts receivable of the Sampaguita Co. shows the following:
Balance, January 1 P24,000
Balance, January 31 25,200
Accounts receivable written off 2,800
Cash sales, January 2,400
Collection on Accounts, January 38,000
Sales allowances for January 400
What amount should be reported for gross sales on the income statement for January? (E)
a. P41,600 c. P66,400
b. P44,800 d. P68,800 RPCPA 1082

Cash collected, decrease in accounts receivables, increase in notes receivable, write-off, sales returns & allowances
19. Herman Halili, who is in business as a sole owner, kept inadequate accounting records. Most of the operations are
summarized in a cash journal. Memorandum entries are made to record non-cash transactions. Upon examination of
available data in 1981 you gathered the following:
1/1/81 12/31/81
Accounts receivable – trade P41,280 P40,520
Notes receivable from customers 20,000 30,000
Accounts payable – trade 38,200 37,300
Cash received from customers 492,480
Accounts receivable written off 2,500
Cash paid to trade creditors 483,000
Sales returns and allowances 2,370
Sales discounts 380
Purchase returns 920
Purchase discounts 400
What is the gross sales for 1981? (E)
a. P501,720 c. P506,970
b. P498,970 d. P496,470 RPCPA 0582

Cash Sales
Total sales – (collection – decrease in accounts receivable) - discounts
16. The total sales for April per books of the Regent Services Corp. amounted to P154,900. The accounts receivable
ledgers showed that April 1 balances were P8,100 and on April 30, P8,000. Cash collected on accounts amounted to
P80,000. P800 were given out as prompt-payment discounts.
What were the cash sales during the month? (M)
a. P74,500 c. P75,000
b. P74,900 d. P74,200 RPCPA 0579

Total sales, decrease in accounts receivable, collection, write-off given


20. These are some of the records of PHILIPPINE Company for the year 1987:
Accounts receivable, January 1, 1987 P210,000
Accounts receivable, December 31, 1987 180,000
Cash received on accounts receivable 320,000
Accounts receivable written-off 7,000
Gross sales in 1987 550,000
The cash sales of PHILIPPINE Company in 1987 is (M)
a. P253,000 c. P193,000
b. P267,000 d. P297,000 RPCPA 1087

Credit Sales
Collection – decrease in A/R
. Beginning and ending Accounts Receivable balances were $28,000 and $24,000, respectively. If collections from clients
during the period were $80,000, then total services rendered on account were apparently (E)
a. $76,000. c. $104,000.
b. $84,000. d. $108,000. S, S & S

Collection from sales


Cash sales, credit sales, increase in receivables given
33. The income statement of ABC Co. shows credit sales of $948. Cash sales for the period were $240. Accounts
receivable on 1/1 were $34; Accounts receivable on 12/31 were $48. Cash collected for sales was (M)
a. $962. c. $934.
b. $1,202. d. $1,174. HGT

Cash sales, credit sales, decrease in receivables given


33. The income statement of ABC Co. shows credit sales of $948. Cash sales for the period were $240. Accounts
receivable on 1/1 were $48; Accounts receivable on 12/31 were $34. Cash collected for sales was (M)
a. $962. c. $934.
b. $1,202. d. $742. L&H

Cash sales, total sales, increase in receivables given


46. The income statement of Viola Co. for 20x1 shows sales of $957. Cash sales for the period were $126. Accounts
receivable on 1/1/x1 were $28; Accounts receivable on 12/31/x1 were $34. Cash collected during 20x1 for sales was
(M)
a. $963. c. $837
b. $951 d. $825 L&H

Cash sales, total sales, decrease in receivables given


46. The income statement of Viola Co. for 20x1 shows sales of $957. Cash sales for the period were $126. Accounts
receivable on 1/1/x1 were $34; Accounts receivable on 12/31/x1 were $28. Cash collected during 20x1 for sales was
(D)
a. $963. c. $837
b. $951 d. $825 L&H

Sales, increase in receivables given


43. Motley, Inc. has sales of $195,000. Accounts receivable on 1/1 were $12,200; accounts receivable on 12/31 were
$14,600. Accounts payable on 1/1 were $12,600; accounts payable on 12/31 were $10,000. Cash collected for sales
was (E)
a. $193,100 c. $192,600
b. $198,100 d. $197,400 L&H

45. The income statement for Primeau Co. shows sales of $21,600. Cost of sales averages 70% of sales. The following
information is also available.
1/1 12/31
Inventory $2,190 $2,750
Accounts payable $1,880 $2,100
Accounts receivable $3,800 $4,600
Cash collected for sales was (M)
a. $15,680 c. $20,800
b. $15,900 d. $22,400 L&H

Sales, decrease in receivables given


42. Staples Inc. has sales of $215,000. Accounts payable on 1/1 were $12,600, accounts receivable on 1/1 were $21,800.
Accounts payable on 12/31 were $14,700, accounts receivable on 12/31 were $19,400. Cash collected for sales was
(E)
a. $215,600 c. $213,200
b. $217,700 d. $217,400 L&H

43. Motley, Inc. has sales of $195,000. Accounts receivable on 1/1 were $14,600; accounts receivable on 12/31 were
$12,200. Accounts payable on 1/1 were $12,600; accounts payable on 12/31 were $10,000. Cash collected for sales
was (E)
a. $193,100 c. $192,900
b. $198,100 d. $197,400 L&H

45. The income statement for Primeau Co. shows sales of $21,600. Cost of sales averages 70% of sales. The following
information is also available.
12/31 1/1
Inventory $2,190 $2,750
Accounts payable $1,880 $2,100
Accounts receivable $3,800 $4,600
Cash collected for sales was (E)
a. $15,680 c. $20,800
b. $15,900 d. $22,400 L&H

Sales, increase in net receivables, write-offs given


. An internal auditor is deriving cash flow data based on an incomplete set of facts. Bad debt expense was 2,000.
Additional data for this period follows:
Net sales 100,000
Accounts receivable beginning balance 5,000
Allowance for bad debts beginning balance (500)
Accounts receivable written off 1,000
Increase in net accounts receivable (after subtraction of allowance for
bad debts) 30,000
How much cash was collected this period? (D)
A. 67,000 C. 68,000
B. 68,500 D. 70,000 CIA 1193 IV-41

10. DELTRA Co.’s year-end financial statement contained the following data:
12/31/89 12/31/90
Merchandise inventory P135,000 P126,000
Trade accounts receivable – net 70,200 75,600
Trade accounts payable 85,500 88,200
Sales 990,000 1,080,000
Cost of goods sold 693,000 756,000
Accounts written off 4,500 4,500
How much cash was collected during 1990 resulting from sales in 1989 and 1990? (D)
a. P1,074,600 c. P1,009,800
b. P1,085,400 d. P1,080,900 HGT

Sales, decrease in net receivables, write-off given


10. DELTRA Co.’s year-end financial statement contained the following data:
12/31/89 12/31/90
Merchandise inventory P135,000 P126,000
Trade accounts receivable – net 75,600 70,200
Trade accounts payable 85,500 88,200
Sales 990,000 1,080,000
Cost of goods sold 693,000 756,000
Accounts written off 4,500 4,500
How much cash was collected during 1990 resulting from sales in 1989 and 1990?
a. P1,074,600 c. P1,009,800
b. P1,085,400 d. P1,080,900 RPCPA 1091

Sales, increase in receivables, increase in unearned revenues given


11. The following figures were taken from the books of PRC Co. Ltd.:
1975 1976
Revenues from operation P199,892 P227,765
Selling expenses 12,879 14,671
Income tax expense 16,872 20,122
Accounts receivable 25,365 30,128
Income taxes payable - 737
Prepaid selling expenses 2,690 2,824
Unearned revenues from operation 4,130 4,855
How much cash did the company receive which was due to revenues from operation during 1976? (D)
a. P228,490 c. P223,727
b. P227,040 d. P195,854 RPCPA 0577

11. The following figures were taken from the books of PRC Co. Ltd.:
1976 1975
Revenues from operation P199,892 P227,765
Selling expenses 12,879 14,671
Income tax expense 16,872 20,122
Accounts receivable 30,128 25,365
Income taxes payable - 737
Prepaid selling expenses 2,690 2,824
Unearned revenues from operation 4,855 4,130
How much cash did the company receive which was due to revenues from operation during 1976? (D)
a. P228,490 c. P223,727
b. P227,040 d. P195,854 HGT

Sales, decrease in receivables, increase in unearned revenues given


11. The following figures were taken from the books of PRC Co. Ltd.:
1976 1975
Revenues from operation P199,892 P227,765
Selling expenses 12,879 14,671
Income tax expense 16,872 20,122
Accounts receivable 25,365 30,128
Income taxes payable - 737
Prepaid selling expenses 2,690 2,824
Unearned revenues from operation 4,855 4,130
How much cash did the company receive which was due to revenues from operation during 1976? (D)
a. P228,490 c. P223,727
b. P227,040 d. P205,380 HGT

11. The following figures were taken from the books of PRC Co. Ltd.:
1975 1976
Revenues from operation P199,892 P227,765
Selling expenses 12,879 14,671
Income tax expense 16,872 20,122
Accounts receivable 30,128 25,365
Income taxes payable - 737
Prepaid selling expenses 2,690 2,824
Unearned revenues from operation 4,130 4,855
How much cash did the company receive which was due to revenues from operation during 1976? (D)
a. P228,490 c. P223,727
b. P227,040 d. P233,253 RPCPA 0577
Sales, increase in receivables, bad debt expense, decrease in allowance for bad debts given
. Barnes Corporation reported the following (in thousands of dollars) for the year:
Beginning Balance Ending Balance
Accounts receivable $600 $850
Allowance for bad debts 40 35
Sales on account were $2,000 and bad debt expense was $10 for the year. How much cash was collected from customers
on account? (M)
A. $1,735. C. $1,745.
B. $1,750. D. $2,250. S, S & T

Sales, decrease in receivables, bad debt expense, decrease in allowance for bad debts given
. Barnes Corporation reported the following (in thousands of dollars) for the year:
Beginning Balance Ending Balance
Accounts receivable $850 $600
Allowance for bad debts 40 35
Sales on account were $2,000 and bad debt expense was $10 for the year. How much cash was collected from customers
on account? (M)
A. $1,735. C. $1,745.
B. $1,750. D. $2,235. HGT

Purchase, Accounts Payable, Payments


Gross Purchases
Cost of sales + increase in inventory
79. Haagen Inc. is a merchandising company. Last month the company's cost of goods sold was $92,000. The company's
beginning merchandise inventory was $14,000 and its ending merchandise inventory was $16,000. What was the total
amount of the company's merchandise purchases for the month? (E)
A. $92,000 C. $122,000
B. $94,000 D. $90,000 G & N 10e

80. Haak Inc. is a merchandising company. Last month the company's cost of goods sold was $62,000. The company's
beginning merchandise inventory was $15,000 and its ending merchandise inventory was $21,000. What was the total
amount of the company's merchandise purchases for the month? (E)
A. $98,000 C. $68,000
B. $62,000 D. $56,000 G & N 10e

. Dave's Duds reported cost of goods sold of $2,000,000 this year. The inventory account increased by $200,000 during
the year to an ending balance of $400,000. What was the cost of merchandise that Dave purchased during the year? (E)
A. $1,600,000. C. $2,200,000.
B. $1,800,000. D. $2,400,000. SS&T

Cost of sales – decrease in inventory


61. Haack Inc. is a merchandising company. Last month the company's cost of goods sold was $84,000. The company's
beginning merchandise inventory was $20,000 and its ending merchandise inventory was $18,000. What was the total
amount of the company's merchandise purchases for the month? (E)
a. $86,000 c. $84,000
b. $82,000 d. $122,000 G & N 9e

81. Haaker Inc. is a merchandising company. Last month the company's cost of goods sold was $87,000. The company's
beginning merchandise inventory was $21,000 and its ending merchandise inventory was $18,000. What was the total
amount of the company's merchandise purchases for the month? (E)
A. $84,000 C. $90,000
B. $126,000 D. $87,000 G & N 10e
141.During the year, Darla’s Pet Shop’s merchandise inventory decreased by $20,000. If the company’s cost of goods sold
for the year was $300,000, purchases must have been
a. $320,000. c. $260,000.
b. $280,000. d. Unable to determine.

. During the year 2008, the inventory of Barbara's Gift Shop decreased by $40,000. If the income statement for the year
2008 reported cost of goods sold of $300,000, purchases during the year must have amounted to:
A. $350,000. C. $260,000.
B. $270,000. D. Some other amount.

Cash payment – decrease in accounts payable + purchase discount


48. The following information is available concerning the accounts of Franz Company:
Accounts payable, January 1, 2001 $18,000
Accounts payable, December 31, 2001 10,000
Cash payments on account during 2001 65,000
Purchase discounts taken during 2001 on 2001 purchases 1,200
Assuming the company records purchases at the gross amounts, the total purchases for 2001 would be (M)
a. $72,200. c. $58,200.
b. $55,800. d. $57,000. K, W & W

. The following information is available concerning the accounts of Franz Company:


Accounts payable, January 1, 2004 $18,000
Accounts payable, December 31, 2004 10,000
Cash payments on account during 2004 75,000
Purchase discounts taken during 2004 on 2004 purchases 1,200
Assuming the company records purchases at the gross amounts, the total purchases for 2004 would be
a. $82,200. c. $68,200.
b. $65,800. d. $57,000.

Cash Paid to Suppliers


Cost of sales + increase in inventory – increase in accounts payable
. Felina Co., a calendar-year enterprise, reported the following information for its most recent fiscal year:
Inventory (January 1) $100,000
Inventory (December 31) 150,000
Accounts payable (January 1) 50,000
Accounts payable (December 3) 60,000
Cost of goods sold 600,000
If payables are incurred solely to acquire inventory, Felina's cash payments to suppliers for inventory must have been (E)
A. $660,000 C. $600,000
B. $640,000 D. $560,000 Gleim

26. The income statement for PQR Co. for 20x1 shows cost of sales of $742. The following information is also available.
12/31 1/1
Inventory $61 $52
Accounts payable $48 $28
Cash paid in 20x1 for merchandise was (M)
a. $713 c. $771
b. $731 d. $779 L&H

Cost of sales + increase in inventory + decrease in accounts payable


3. Following information is available for Asian Co. for the month of September:
Accounts payable, September 1 P110,000
Accounts payable, September 30 130,000
Inventory, September 1 90,000
Inventory, September 30 80,000
Cost of goods sold in September 450,000
The cash disbursements to trade creditors in September were (E)
a. P450,000 c. P420,000
b. P440,000 d. Answer not given. RPCPA 1080

35. The income statement for Rawlye Co. shows cost of sales of $835. The following information is also available.
1/1 12/31
Inventory $52 $71
Accounts payable $58 $28
Cash paid for merchandise was (E)
a. $786 c. $884
b. $805 d. $824 L&H

. Dooling Corporation reported balances in the following accounts for the current year:
Ending Beginning
Inventories $600 $300
Accounts payable 300 500
Cost of goods sold was $7,500. What was the amount of cash paid to suppliers? (E)
A. $7,000. C. $7,300.
B. $7,200. D. $8,000. S, S & T

27. The income statement of Scoot Co. shows cost of sales of $722. The following data are also available.
1/1 12/31
Inventory $ 68 $ 94
Accounts receivable $130 $101
Accounts payable $ 54 $ 31
Cash paid for merchandise was (E)
a. $673. c. $719.
b. $683. d. $771. L&H

36. The income statement of Radke Co. shows cost of sales of $1,076. The following data are also available.
12/31 1/1
Inventory $128 $ 84
Accounts receivable $130 $145
Accounts payable $ 31 $ 54
Cash paid for merchandise was (E)
a. $1,120 c. $1,135
b. $1,143 d. $1,091 L&H

49. The income statement of Hull Co. shows cost of sales of $192,760. The following data are also available.
12/31 1/1
Inventory $28,650 $22,580
Accounts receivable $13,600 $14,540
Accounts payable $33,610 $35,440
Cash paid for merchandise was (E)
a. $188,520 c. $197,890
b. $200,660 d. $198,830 L&H
9. DELTRA Co.’s year-end financial statement contained the following data:
12/31/89 12/31/90
Merchandise inventory P135,000 P126,000
Trade accounts receivable – net 75,600 70,200
Trade accounts payable 85,500 88,200
Sales 990,000 1,080,000
Cost of goods sold 693,000 756,000
Accounts written off 4,500 4,500
How much cash was disbursed during 1990 for purchase of merchandise? (E)
a. P879,300 c. P744,300
b. P832,500 d. P747,000 RPCPA 1091

Cost of sales – decrease in inventory – increase in accounts payable


26. The income statement for PQR Co. for 20x1 shows cost of sales of $742. The following information is also available.
1/1 12/31
Inventory $61 $52
Accounts payable $48 $28
Cash paid in 20x1 for merchandise was (E)
a. $713 c. $753
b. $731 d. $779 HGT

35. The income statement for Rawlye Co. shows cost of sales of $835. The following information is also available.
12/31 1/1
Inventory $52 $71
Accounts payable $58 $28
Cash paid for merchandise was (E)
a. $786 c. $854
b. $805 d. $824 L&H

. Dooling Corporation reported balances in the following accounts for the current year:
Beginning Ending
Inventories $600 $300
Accounts payable 300 500
Cost of goods sold was $7,500. What was the amount of cash paid to suppliers? (E)
A. $7,000. C. $7,300.
B. $7,200. D. $7,500. S, S & T

27. The income statement of Scoot Co. shows cost of sales of $722. The following data are also available.
12/31 1/1
Inventory $ 68 $ 94
Accounts receivable $130 $101
Accounts payable $ 54 $ 31
Cash paid for merchandise was (E)
a. $673. c. $719.
b. $683. d. $725. L&H

36. The income statement of Radke Co. shows cost of sales of $1,076. The following data are also available.
1/1 12/31
Inventory $128 $ 84
Accounts receivable $130 $145
Accounts payable $ 31 $ 54
Cash paid for merchandise was (E)
a. $1,120 c. $1,135
b. $1,143 d. $1,009 L&H

49. The income statement of Hull Co. shows cost of sales of $192,760. The following data are also available.
1/1 12/31
Inventory $28,650 $22,580
Accounts receivable $13,600 $14,540
Accounts payable $33,610 $35,440
Cash paid for merchandise was (E)
a. $184,860 c. $197,890
b. $200,660 d. $198,830 HGT

9. DELTRA Co.’s year-end financial statement contained the following data:


12/31/89 12/31/90
Merchandise inventory P135,000 P126,000
Trade accounts receivable – net 75,600 70,200
Trade accounts payable 85,500 88,200
Sales 990,000 1,080,000
Cost of goods sold 693,000 756,000
Accounts written off 4,500 4,500
How much cash was disbursed during 1990 for purchase of merchandise? (E)
a. P879,300 c. P744,300
b. P832,500 d. P747,000 RPCPA 1091

3. Following information is available for Asian Co. for the month of September:
Accounts payable, September 1 P130,000
Accounts payable, September 30 110,000
Inventory, September 1 80,000
Inventory, September 30 90,000
Cost of goods sold in September 450,000
The cash disbursements to trade creditors in September were (E)
a. P450,000 c. P420,000
b. P440,000 d. P480,000 HGT

Cost of sales – decrease in inventory + decrease in accounts payable


. Felina Co., a calendar-year enterprise, reported the following information for its most recent fiscal year:
Inventory (January 1) $150,000
Inventory (December 31) 100,000
Accounts payable (January 1) 60,000
Accounts payable (December 3) 50,000
Cost of goods sold 600,000
If payables are incurred solely to acquire inventory, Felina's cash payments to suppliers for inventory must have been
(E)
A. $660,000 C. $600,000
B. $640,000 D. $560,000 Gleim

(Purchases – discount) – increase in accounts payable


6. At the beginning of the current year there was a P20,000 balance in accounts payable; at the end of the year accounts
payable balance was P25,000. During the year purchases of P150,000 were made; some of those invoices were of
course unpaid at year-end. All purchases are subject to terms of 2/10, n/30. On the invoices paid during the year all
discounts were taken. Considering these facts, cash paid to creditors during the year must have been (D)
a. P145,000 c. P145,100
b. P147,000 d. P142,100 RPCPA 1075

Cost of sales, decrease in inventory, increase in payables, purchase returns & discount
4. You are given the following information relating to Violet Co.
Accounts payable trade, January 1 P28,400
Accounts payable trade, December 31 32,200
Inventory, January 1 62,700
Inventory, December 31 59,800
Cost of goods sold for the year 316,600
Purchase discount 7,600
Purchase returns 9,600
Assuming all purchases were on account, how much was paid in total to trade creditors during the year? (M)
a. P292,800 c. P313,700
b. P309,900 d. P292,700 RPCPA 1077

Cost of goods sold


Payment + increase in payable – increase in inventory
. The following information is available for Ace Company for 2004:
Disbursements for purchases $700,000
Increase in trade accounts payable 50,000
Increase in merchandise inventory 20,000
Costs of goods sold for 2004 was (M)
a. $770,000. c. $670,000.
b. $730,000. d. $630,000. CPA Adapted

166.The following information is available for Ace Company for 2010:


Disbursements for purchases $1,050,000
Increase in trade accounts payable 75,000
Increase in merchandise inventory 30,000
Costs of goods sold for 2010 was
a. $1,155,000. c. $1,005,000.
b. $1,095,000. d. $945,000.

4. Following information are taken from the books of Sterling Co. for the month of April, 1981:
April 30 April 1
Inventory P 40,000 P35,000
Accounts payable 56,000 48,000
Cash paid on accounts payable was P169,900 and purchase returns was P2,100.
The cost of goods sold for April, 1981 amounted to (M)
a. P175,000 c. P180,000
b. P172,900 d. Answer not given RPCPA 0581

Payment + increase in payable + decrease in inventory


28. The following information is available for Dayrit Company for 1995:
Disbursements for purchases of merchandise P290,000
Decrease in merchandise inventory 10,000
Increase in trade accounts payable 25,000
How much is the cost of goods sold for 1995? (M)
a. P255,000 c. P305,000
b. P275,000 d. P325,000 S, S & S, RPCPA 1095

61. The following information is available for Ace Company for 2001:
Disbursements for purchases $500,000
Decrease in merchandise inventory 20,000
Increase in trade accounts payable 50,000
Costs of goods sold for 2001 was
a. $570,000. c. $470,000.
b. $530,000. d. $430,000. K, W & W

55. The following information is available for Avalon Company for 2004:
Disbursements for purchases of merchandise ........... 580,000
Decrease in merchandise inventory .................... $ 20,000
Increase in accounts payable related to inventory .... 50,000
What amount should Avalon report as cost of goods sold for 2004? (M)
a. $510,000 c. $610,000
b. $550,000 d. $650,000 S, S & S

. The following information is available for Ace Company for 2004:


Disbursements for purchases $700,000
Increase in trade accounts payable 50,000
Decrease in merchandise inventory 20,000
Costs of goods sold for 2004 was
a. $770,000. c. $670,000.
b. $730,000. d. $630,000. CPA Adapted

166.The following information is available for Ace Company for 2010:


Disbursements for purchases $1,050,000
Increase in trade accounts payable 75,000
Decrease in merchandise inventory 30,000
Costs of goods sold for 2010 was
a. $1,155,000. c. $1,005,000.
b. $1,095,000. d. $945,000.

. The following balances were reported by Oland Co. at December 31, 2001 and 2000:
12/31/01 12/31/00
Inventory $260,000 $290,000
Accounts payable 75,000 50,000
Oland paid suppliers $490,000 during the year ended December 31, 2001. What amount should Oland report for cost of
goods sold in 2001? (E)
a. $545,000 c. $485,000
b. $495,000 d. $435,000 AICPA 0592

24. Following are the account balances at December 31, 1996 and 1995 of Lille Company:
12/31/96 12/31/95
Inventory 520,000 580,000
Accounts payable 150,000 100,000
Lille paid P980,000 to its creditors during the year ended December 31, 1996. (E)
Lille should show as cost of goods sold in 1996 the amount of
a. P870,000 c. P990,000
b. P1,090,000 d. P970,000 RPCPA 1096

38. The following balance were reported by Union Co. at December 31, 1998 and 1997:
12/31/98 12/31/97
Inventory P2,600,000 P2,900,000
Accounts payable 750,000 500,000
Union paid suppliers P4,900,000 during the year ended December 31, 1998.
What amount should Union report for cost of goods sold in 1998? (E)
a. P5,450,000 c. P4,950,000
b. P4,350,000 d. P4,850,000 RPCPA 0598

Payment – decrease in payable – increase in inventory


. The following information is available for Ace Company for 2004:
Disbursements for purchases $700,000
Decrease in trade accounts payable 50,000
Increase in merchandise inventory 20,000
Costs of goods sold for 2004 was
a. $770,000. c. $670,000.
b. $730,000. d. $630,000. CPA Adapted

166.The following information is available for Ace Company for 2010:


Disbursements for purchases $1,050,000
Decrease in trade accounts payable 75,000
Increase in merchandise inventory 30,000
Costs of goods sold for 2010 was
a. $1,155,000. c. $1,005,000.
b. $1,095,000. d. $945,000.

Payment – decrease in payable + decrease in inventory


. The following information is available for Ace Company for 2004:
Disbursements for purchases $700,000
Decrease in trade accounts payable 50,000
Decrease in merchandise inventory 20,000
Costs of goods sold for 2004 was (M)
a. $770,000. c. $670,000.
b. $730,000. d. $630,000. CPA Adapted

166.The following information is available for Ace Company for 2010:


Disbursements for purchases $1,050,000
Decrease in trade accounts payable 75,000
Decrease in merchandise inventory 30,000
Costs of goods sold for 2010 was
a. $1,155,000. c. $1,005,000.
b. $1,095,000. d. $945,000.

49. The following information is available for Carr Company:


12/31/01 1/1/01
Inventory $9,700 $10,400
Accounts payable 7,200 9,000
Payments for goods during 2001 was $62,000. Cost of goods sold for 2001 is (E)
a. $59,500. c. $67,100.
b. $60,900. d. $68,500. K, W & W

. The following information is available for Carr Company:


12/31/01 1/1/01
Inventory $9,700 $10,400
Accounts payable 7,200 9,000
Payment for goods during 2004 was $82,000. Cost of goods sold for 2004 is
a. $79,500. c. $87,100.
b. $80,900. d. $88,500.

Revenue & Expenditure Cycle - Inventory


Accounts Receivable, Ending balance
Collections, purchases, ending inventory, markup ratio given
. Steven Corporation began operations in 2010. For the year ended December 31, 2010, Steven made available the
following information:
Total merchandise purchases for the year $350,000
Merchandise inventory at December 31, 2010 70,000
Collections from customers 200,000
All merchandise was marked to sell at 40% above cost. Assuming that all sales are on a credit basis and all receivables
are collectible, what should be the balance in accounts receivable at December 31, 2010? (M)
A: $ 50,000 C: $250,000
B: $192,000 D: $290,000 Wiley 2011

. Steven Corporation began operations in 2004. For the year ended December 31, 2004 Steven made available the following
information:
Total merchandise purchases for the year 7,000,000
Merchandise inventory at December 31 1,400,000
Collection from customers 4,000,000
All merchandise was marked to sell at 40% above cost. Assuming that all sales are on a credit basis and all receivables are
collectible, what should be the balance in accounts receivable at December 31, 2004? (M)
A. 1,000,000 C. 5,000,000
B. 3,840,000 D. 5,800,000 AICPA

Beginning receivables, collections, COGAS, ending inventory, gross profit ratio given
3. You are given the following information relating to Palad Trading, a general merchandising company:
Rate of gross profit on sales 20%
Accounts receivable, December 31, 1980 P 80,000
Collections on accounts receivable in 1981 430,000
Cost of goods available for sale in 1981 460,000
Merchandise inventory, December 31, 1981 100,000
Assuming all sales were on account, what was the company’s accounts receivable balance on December 31, 1981? (M)
a. P100,000 c. P120,000
b. P50,000 d. P90,000 RPCPA 0582

2. Following are data pertaining to a trading company:


Total cost of goods available for sale in 1976 P46,000
Inventory, December 31, 1976 10,000
Accounts receivable, December 31, 1975 9,000
Collections on accounts receivable in 1976 45,000
Gross profit (margin) on sales 25%
What was the company’s accounts receivable balance on December 31, 1976? (M)
a. P6,000 c. P10,000
b. P9,000 d. P12,000 RPCPA 1077

Cash sales, gross profit, beg. receivables, collection on account, purchases, decrease in nventory given
7. The following are the data pertaining to the 1977 operations of the GHI Company:
Cash Sales P 7,500
Gross Profit on sales (gross margin) 13,500
Accounts receivable, Jan. 1, 1977 12,000
Accounts receivable collected during 1977 39,000
Purchases of Inventory during 1977 30,000
Inventory, Jan. 1, 1977 18,000
Inventory, Dec. 31, 1977 16,500
The accounts receivable balance at December 31, 1977 is (M)
a. P10,500 c. P19,500
b. P18,000 d. P25,500 RPCPA 1078

2. Certain information relative to the 1981 operation of the Angel Co. follows:
Cash sales during 1981 50,000
Gross profit on sales (gross margin) 90,000
Accounts receivable, January 1, 1981 P80,000
Accounts receivable collected during 1981 260,000
Purchases during 1981 200,000
Inventory, January 1, 1981 120,000
Inventory, December 31, 1981 110,000
What is Angel’s accounts receivable balance at December 31, 1981? (M)
a. P70,000 d. P170,000
b. P120,000 e. P250,000
c. P130,000 RPCPA 1082

. Certain information relative to the 2004 operations of Foley Company follows:


Cash sales 1,000,000
Gross profit on sales 1,800,000
Accounts receivable, January 1 1,600,000
Accounts receivable collected 5,200,000
Purchases 4,000,000
Inventory January 1 2,400,000
Inventory, December 31 2,200,000
What is Foley's accounts receivable balance at December 31, 2004? (M)
A. 1,400,000 C. 3,400,000
B. 2,400,000 D. 2,600,000 AICPA

Beginning receivables, credit sales, sales return, write-off, collections, estimated future sales returns, estimated uncollectible
accounts given
31. The following information relates to Jayson Co.’s accounts receivable for 1995”
Accounts receivable, 1/1/95 P 650,000
Credit sales for 1995 2,700,000
Sales return for 1995 75,000
Accounts written off during 1995 40,000
Collections from customers during 1995 2,150,000
Estimated future sales returns at 12/31/95 50,000
Estimated uncollectible accounts at 12/31/95 110,000
What amount should Jayson report for accounts receivable before allowances for sales returns and uncollectible
accounts, at December 31, 1995? (M)
a. P1,200,000 c. P1,085,000
b. P1,125,000 d. P925,000 RPCPA 1095

Inventory, Beginning Balance


11. Following information pertain to Prudential Company for the first quarter of 1981
Cash sales 32,000
Gross profit rate on sales 30%
Accounts receivable, Jan. 1, 1981 P 90,000
Accounts receivable, Mar. 31, 1981 85,000
Cash collected on accounts receivable 220,000
Bad debts written off 3,500
Purchases, net 180,350
Inventory, March 31, 1981 52,000
The inventory on January 1, 1981 is (M)
a. P24,600 c. P52,000
b. P47,000 d. Answer not given RPCPA 0581

Inventory, Ending Balance


Cost of Goods Available for Sale, Cost of Goods Sold
90. From the following information, determine the amount of ending inventory. (E)
Beginning Inventory ................................... $20,000
Purchases ............................................. 41,000
Freight-In ............................................ ?
Purchase Returns and Allowances ....................... 3,000
Purchase Discounts .................................... 4,000
Cost of Goods Available for Sale ...................... 55,000
Ending Inventory ...................................... ?
Cost of Goods Sold .................................... 22,000
A. $23,000 C. $33,000
B. $32,000 D. $22,000 S, S & S

Beg inventory, purchases, collection, beg & ending A/R, markup, given
30. Brad Company prepared monthly income statements. A physical inventory is taken only at year-end, hence month-end
inventories must be estimated. All sales are made on account. The rate of markup on cost is 50%. The following
information relates to the month of September:
Accounts receivable, September 1 P100,000
Accounts receivable, September 30 150,000
Collection of accounts receivable during September 250,000
Inventory, September 1 180,000
Purchases of inventory during September 160,000
The estimated cost of September 30 inventory is (M)
a. P120,000 c. P190,000
b. P140,000 d. P220,000 RPCPA 1095

. Finland Co. prepares monthly income statements. A physical inventory is taken only at year-end; hence, month-end
inventories must be estimated. All sales are made on account. The rate of markup on cost is 50%. The following
information related to the month of June:
Accounts receivable, June 1 $20,000
Accounts receivable, June 30 30,000
Collection of accounts receivable during June 50,000
Inventory, June 1 36,000
Purchase of inventory during June 32,000
The estimated cost of the June 30 inventory is (M)
a. $24,000 c. $38,000
b. $28,000 d. $44,000 AICPA 1173 I-10

15. The YVC Corp. prepares monthly income statements. As a cost saving device, a physical inventory is taken only at
year-end, hence, month-end inventories must be estimated. All sales are made on account. The rate of markup on cost
is 50%.
Accounts receivable, March 1, 1980 60,000
Accounts receivable, March 31, 1980 90,000
Collection of accounts receivable 150,000
Inventory, March 1, 1980 P108,000
Purchase of inventory 96,000
The estimated cost of the March 31, 1980 inventory is (M)
a. P79,000 c. P88,000
b. P84,000 d. P93,000 RPCPA 0580

40. Jupiter Company prepares monthly income statements. A physical inventory is taken only at year-end; hence, month-
end inventories must be estimated. All sales are made on account. The rate of markup on cost is 50 percent. The
following information relates to the month of May:
Accounts receivable, May 1 ............................ $20,000
Accounts receivable, May 31 ........................... 30,000
Collection of accounts receivable during May .......... 50,000
Inventory, May 1 ...................................... 36,000
Purchases of inventory during May ..................... 32,000
The estimated cost of the May 31 inventory is (M)
a. $24,000. c. $38,000.
b. $28,000. d. $44,000. S, S & S

Sales, gross profit, beginning inventory, purchases given


. The following amounts relate to an enterprise:
Sales 180,000
Gross margin 60,000
Beginning inventory 50,000
Purchases 170,000
The amount of ending inventory is (E)
A. 60,000 C. 120,000
B. 100,000 D. 160,000 CIA 0597 IV-11

58. Delta Merchandising, Inc., has provided the following information for the year just ended:
Net sales $128,500
Gross margin 38,550
Beginning inventory 24,000
Purchases 80,000
The ending inventory for the company at year end was: (E)
a. $65,450. c. $14,050.
b. $24,500. d. $9,950. G & N 9e

Sales, gross profit ratio, beginning inventory, purchases given


3. The merchandise inventory of CD Ltd. On January 1, 1988 was P450,000. During 1988, the company has recorded
sales of P1,500,000 and merchandise purchases of P1,100,000. The gross profit (margin) on these sales was 30%.
What is the merchandise inventory as at December 31, 1988?
a. P500,000 c. P1,100,000
b. P650,000 d. Answer not given. RPCPA 1089

7. From the information below for the period July 1 to September 30, compute the estimated inventory at September 30:
Sales, net, for the period 440,000
Gross profit (margin) on sales 35%
Inventory, July 1 P 45,000
Purchases, net, for the period 300,000
a. P95,000 c. P54,000
b. P59,000 d. None of these. RPCPA 1077

Cash sales, collection from A/R, decrease in receivables, write-off, purchases, ending inventory, gross profit ratio given
5. Following information pertain to Angeles Co. for the year 1982
Cash sales P 64,000
Cash collected on accounts receivable 440,000
Accounts receivable, December 31, 1981 110,000
Accounts receivable, December 31, 1982 95,000
Bad debts written off 6,500
Purchases (net) during 1982 350,200
Inventory. December 31, 1982 84,000
Gross profit on sales 30%
The company’s inventory on December 31, 1981 was (M)
a. P64,000 c. P84,000
b. P80,650 d. None of these RPCPA 0583

Increase in receivables, AR turnover, beg inventory, purchases, gross profit ratio given
6. Following data pertain to Sterling Corporation:
Accounts receivable, December 31, 1977 P55,000
Accounts receivable, June 30, 1978 65,000
Accounts receivable turnover 5 to 1
Inventory, December 31, 1977 90,000
Purchases, January to June, 1978 225,000
Average gross profit rate 33 – 1/3%
The inventory at June 30, 1978 based on the above data was (E)
a. P115,000 c. P75,000
b. P105,000 d. P30,000 RPCPA 1078

Gross Purchases
2. Following data relate to Orient Co., a trading firm, for the year 1979:
Collections on accounts receivable P180,000
Accounts receivable, Jan. 1, 1979 36,000
Accounts receivable, Dec. 31, 1979 48,000
Inventory, Jan. 1, 1979 44,000
Inventory, Dec. 31, 1979 40,000
Gross profit rate on sales 25%
The company’s purchases for 1979 were (E)
a. P140,000 c. P184,000
b. P180,000 d. P68,000 RPCPA 1080

Cash Paid to Suppliers


Sales, cost of sales ratio, increase in inventory & increase in accounts payable given
44. The income statement for Primeau Co. shows sales of $21,600. Cost of sales averages 70% of sales. The following
information is also available.
1/1 12/31
Inventory $2,190 $2,750
Accounts payable $1,880 $2,100
Accounts receivable $3,800 $4,600
Cash paid for merchandise was (E)
a. $15,460 c. $14,780
b. $15,900 d. $20,800 HGT
Sales, cost of sales ratio, increase in inventory & decrease in accounts payable given
40. The income statement for Smithson Co. shows sales of $1,200. Cost of sales averages 75% of sales. The following
information is also available.
12/31 1/1
Inventory $292 $275
Accounts payable $ 88 $110
Cash paid for merchandise was (E)
a. $917 c. $635
b. $1,239 d. $939 L&H

Sales, cost of sales ratio, decrease in inventory & increase in accounts payable given
40. The income statement for Smithson Co. shows sales of $1,200. Cost of sales averages 75% of sales. The following
information is also available.
1/1 12/31
Inventory $292 $275
Accounts payable $ 88 $110
Cash paid for merchandise was (E)
a. $635 c. $939
b. $861 d. $1,239 HGT

Sales, cost of sales ratio, decrease in inventory & decrease in accounts payable given
44. The income statement for Primeau Co. shows sales of $21,600. Cost of sales averages 70% of sales. The following
information is also available.
12/31 1/1
Inventory $2,190 $2,750
Accounts payable $1,880 $2,100
Accounts receivable $3,800 $4,600
Cash paid for merchandise was (E)
a. $15,680 c. $14,780
b. $15,900 d. $20,800 L&H

Gross Sales & Purchases


Questions 10 and 11 are based on the following information. RPCPA 0591
The following are found in the accounting records of XYZ Corporation:
Average merchandise inventory P 265,000
Accounts receivable, January 1, 1990 375,500
Merchandise inventory, December 31, 1990 320,000
Accounts receivable, December 31, 1990 345,500
Freight in 40,000
Cash sales 650,000
Purchases returns and allowances 50,000
Cash collected from accounts receivable 1,350,000
Cost of sales 1,530,000

10. The gross sales in 1990 is


a. P1,320,000 c. P1,350,000
b. P650,000 d. P1,970,000

11. The gross purchases in 1990 is


a. P1,650,000 c. P1,640,000
b. P1,690,000 d. P1,970,000
Questions 9 and 10 are based on the following information. RPCPA 1085
Ruby Co. is engaged in a small export business. The company maintains limited records. Most of the company’s
transactions are summarized in a cash journal; non-cash transactions, on the other hand are recorded by making
memorandum entries. The following balances are abstracted from the company’s records:
Jan. 1, 1984 Dec. 31 1984
Accounts receivable P1,500 P1,000
Accounts payable 2,000 1,000
Accounts written off as uncollectible 50
Cash received from customers 21,000
Cash paid to trade creditors 14,000
Furniture and equipment 2,500
Sales discounts 40
Sales returns and allowances 100
Salesmen’s commissions 1,500
Notes receivable (from customers) 500 1,000
Purchase discounts 25
Purchase returns 50
Organizational costs 2,500

9. Gross sales for 1984 is


a. P21,190 c. P21,050
b. P20,550 d. P21,000

10. Gross purchases for 1984 is


a. P13,000 c. P12,000
b. P13,075 d. P12,075

Gross Income
21. Assume that A had an opening inventory of P10,000; he purchased goods for resale during the year at a total cost of
P16,000; the year’s total sales were P30,000; and the closing inventory was P6,000. The cost of merchandise
withdrawn by A for his personal use was P2,000. A’s gross income for the year is (M)
a. P12,000 c. P16,000
b. P14,000 d. None of the above. RPCPA 0578

Retained Earnings
73. From the following information, determine the ending balance in Retained Earnings.
1. Beginning Retained Earnings $ 6,200
2. Cash 1,900
3. Accounts Payable 1,100
4. Sales 27,000
5. Merchandise Inventory 9,200
6. Cost of Goods Sold 14,400
7. Salary Expense 9,900
a. $ 6,200 d. $18,900
b. $ 8,900 e. $20,000
c. $12,700

You might also like